4020 Exam 2 Practice Questions

Réussis tes devoirs et examens dès maintenant avec Quizwiz!

A postpartum patient who had an episiotomy is being discharged with the newborn. Which patient statement should indicate that additional discharge teaching is required? A. "I can go off of pain medication as soon as I get home." B. "I may have less interest in sex." C. "I could experience depression." D. "I may have problems breastfeeding."

A. "I can go off of pain medication as soon as I get home." Episiotomies are painful, and the patient should remain on pain medication until the healthcare provider discontinues the therapy. The pain from an episiotomy has been linked to breastfeeding issues, sexual dysfunction, and depression.

A patient who received a spinal block is experiencing apnea, pupil dilation, and an unobtainable blood pressure. Which action should the nurse take first? A. Establish the airway. B. Administer ephedrine. C. Completely open the intravenous fluid infusion. D. Administer oxygen

A. Establish the airway. The patient is experiencing total spinal block and establishing the airway is the priority. The next action is to administer oxygen, followed by administering ephedrine. Completely opening the intravenous fluid infusion is not indicated at this time. However, if hypotension occurs, IV fluids should be increased.

The nurse is caring for a patient in labor. For which fetal position should the patient be placed on the hands and knees? A. Occiput posterior B. Occiput anterior C. Breech D. Shoulder presentation

A. Occiput posterior When the fetus is in the persistent occiput posterior position, side-lying or hands and knees positions can promote fetal head rotation. Occiput anterior fetal position is not aided by a hands and knees position. A fetus in breech or shoulder presentation may need a cesarean birth.

The nurse assesses a newborn to determine the Apgar score. Which finding should require the newborn to be stabilized? A. Apgar of 10 at 5 minutes B. Apgar of 6 at 1 minute C. Apgar of 8 at 1 minute D. Apgar of 9 at 5 minutes

B. Apgar of 6 at 1 minute The newborn with an Apgar score that is less than 8 at 1 minute, or less than 9 at 5 minutes, requires stabilization rather than remaining with the parents in the birth area.

A patient is having an amnioinfusion. Which outcome should the nurse add to this patient's plan of care? A. The patient experiences no fear until delivery. B. The fetus remains uncompromised during labor and birth. C. The amniotic fluid increases to 10 cm. D. The patient experiences no pain during the procedure.

B. The fetus remains uncompromised during labor and birth. The patient who is experiencing oligohydramnios requires an amnioinfusion to increase the amount of amniotic fluid. Oligohydramnios can compromise the safety of the fetus, so an outcome that addresses this is appropriate. The patient having an amnioinfusion would not experience pain or fear. Amniotic fluid doubling is not an appropriate outcome.

A patient at 32 weeks' gestation is experiencing preterm premature rupture of membranes (PPROM). Which patient statement should indicate that further teaching about PPROM is required? A. "Corticosteroids and antibiotics should be taken to mature the fetus." B. "The UTI I had may have increased my risk for this condition." C. "If natural labor does not start soon, I will need to be induced." D. "I will likely deliver within the next 4 days."

C. "If natural labor does not start soon, I will need to be induced." Patients who experience PPROM after 34 weeks' gestation will benefit from labor induction if natural labor does not immediately begin. A UTI is a risk factor for PPROM. Most patients with PPROM will deliver within 96 hours, or 4 days. Expectant management is appropriate when the patient is at less than 34 weeks' gestation, and antenatal corticosteroids and antibiotics should be given to attempt to gain fetal maturation to viability.

The fetus of a patient in labor is experiencing bradycardia. Which situation should the nurse expect to occur if the bradycardia continues? A. Hypertension B. Delayed delivery C. Acidemia D. Congenital malformations

C. Acidemia If bradycardia is not corrected quickly, the fetus can develop acidemia. Bradycardia does not lead to hypertension, delayed delivery, or congenital malformations.

The nurse reviews a pregnant patient's record and notices cephalic presentation on the most recent sonogram. Which statement should the nurse make? A. "The baby's hips and knees are flexed." B. "The baby's buttocks are presenting." C. "The baby's hips and legs are extended." D. "The baby's head is in complete extension."

D. "The baby's head is in complete extension." A fetus whose head is in complete extension is in cephalic presentation. The other positions are all indicative of breech presentation.

A patient in the first stage of labor is diagnosed with preeclampsia. Which patient statement should indicate the need for further teaching about fetal heart rate assessment? A. "When you listen to my baby's heartbeat, you're going to take my pulse." B. "If you can't find the baby's heartbeat, you're going to move the Doppler until you hear it." C. "You're going to put gel on my belly so you can hear the baby's heartbeat clearly." D. "You're going to listen to my baby's heartbeat every 5 minutes."

D. "You're going to listen to my baby's heartbeat every 5 minutes." Fetal heart rate should be monitored every 15 minutes for a high-risk patient in the first stage of labor. The other patient statements are correct.

The fetus of a patient in labor has a normal baseline heart rate. Which finding should the nurse document? A. 95 beats/min over a 10-minute period B. 165 beats/min over a 10-minute period C. Undulations between 120 and 160 beats/min with variability amplitude of 5 to 15 beats/min D. 120 beats/min between contractions

D. 120 beats/min between contractions Baseline fetal heart rate (FHR) refers to the average FHR that is not associated with uterine contractions. The normal baseline rate ranges from 110 to 160 beats/min. A rate less than 110 beats/min over a 10-minute period indicates fetal bradycardia. A rate greater than 160 beats/min over a 10-minute period indicates fetal tachycardia. A sinusoidal FHR pattern undulates in a sine pattern between 120 and 160 beats/min with variability amplitude of 5 to 15 beats/min.

The nurse is preparing teaching about transvaginal pudendal blocks. Which information should be included regarding nursing care? A. The patient should be continuously monitored. B. Assessment can be delegated to the nursing assistant. C. The nurse only needs to enter the patient's room when equipment alarms sound D. Additional assessments are not necessary.

D. Additional assessments are not necessary. The use of a pudendal block does not alter the need to monitor maternal vital signs or fetal heart rate, so assessments in addition to the expected ones are not necessary. Continuous monitoring is not necessary, but the patient should not be left unattended. The monitoring of a patient in labor should not be delegated to a nursing assistant.

The nurse is caring for patient in preterm labor. Which secondary prevention strategy should the nurse implement? A. Obtain a clean catch urine specimen. B. Refer the patient to a dentist. C. Administer progesterone. D. Administer antibiotics.

D. Administer antibiotics. Secondary prevention strategies for preterm labor include antibiotic therapy to prevent urinary and vaginal infections that occur during pregnancy. Administering progesterone, obtaining a clean catch urine specimen, and referring the patient to a dentist to determine if gum disease is present are all primary prevention strategies.

The nurse notes late decelerations in the fetus of a patient in labor. Which intervention should the nurse question? A. Repositioning patient fully on the side B. Increasing fluids C. Administering 10 L oxygen D. Administering a narcotic

D. Administering a narcotic Systemic analgesics, such as a narcotic, should only be administered if there are no late decelerations or recurrent variable decelerations. Repositioning the patient on the left side, increasing fluids, and administering 10 L oxygen are appropriate interventions for the patient who has hypotension, which causes late decelerations in the fetus.

The nurse is planning care for a patient in labor. For which procedure should care be focused to prevent a prolapsed umbilical cord during labor and delivery? A. Amniocentesis B. Cesarean birth C. Nonstress test D. Amniotomy

D. Amniotomy The risk of a prolapsed umbilical cord is increased once the membranes have ruptured after an amniotomy, especially if the fetal presenting part is not firmly pressed down against the cervix. There is no risk of a prolapsed cord with an amniocentesis, cesarean birth, or a nonstress test.

A patient in labor who is 7 cm dilated reports excruciating pain but worries that an epidural will hurt the baby. Which response should the nurse make? A. "An epidural is unlikely to affect the baby's heart rate." B. "The baby will experience low blood pressure after birth." C. "The epidural will make the baby lethargic, but this will pass quickly." D. "Because an epidural was not planned, it cannot be done at this time."

A. "An epidural is unlikely to affect the baby's heart rate." Epidurals do not cause hypotension in the newborn. An epidural can cause maternal hypotension, which can be minimized by hydrating the vascular system with 500 to 1000 mL of IV solution before an epidural procedure and changing the patient's position and/or increasing the IV rate afterward. It is rare for a continuous epidural to cause sedation in the infant. An epidural can be given regardless of whether or not it was planned.

The nurse teaches a patient about the induction of labor. Which patient statement should indicate the teaching was effective? A. "I need to have labor induced because I have high blood pressure." B. "I need to have labor induced because of my history of placenta previa." C. "I need to have labor induced because the umbilical cord is showing." D. "I need to have labor induced because my baby is breech."

A. "I need to have labor induced because I have high blood pressure." The induction of labor is indicated for a pregnant patient with hypertension. The induction of labor is contraindicated in placenta previa, breech presentation, and prolapsed umbilical cord.

The nurse prepares to assess the fetal heart rate of a patient in labor. Which statement should the nurse make to explain the procedure? A. "I'm going to listen to the heartbeat for a full minute, and I'll need to continue during and after your next contraction." B. "I'm going to wait until your next contraction ends, and then I'll listen to the heartbeat for 15 seconds." C. "I'm going to listen to the heartbeat until your next contraction starts." D. "I'm going to listen to the heartbeat for 45 seconds as soon as your next contraction starts."

A. "I'm going to listen to the heartbeat for a full minute, and I'll need to continue during and after your next contraction." After the fetal heart rate is located, the rate should be assessed for 30 to 60 seconds to obtain the number of beats per minute. The rate should be assessed before, during, and just after a contraction to detect any abnormalities in heart rate. Counting for 15 seconds is not long enough. Stopping the count when a contraction starts and waiting for the contraction before starting the count are not ideal.

A patient in labor is being prepared for a lumbar epidural. Which instruction should the nurse provide to the patient? A. "Round out the small of your back and place your chin on your chest." B. "Increase the curve of your lower back and raise your head." C. "Sit up and extend your legs straight out in front." D. "Curl up in a ball, bringing your knees to your chest."

A. "Round out the small of your back and place your chin on your chest." To facilitate the insertion of the catheter for a lumbar epidural, the patient should be positioned on the left or right side at the edge of the bed with legs slightly flexed, or may be asked to sit on the edge of the bed. The patient should also be instructed to drop the shoulders, round out the small of the back, and drop the chin to the chest.

The nurse is assessing the emotional health of a patient receiving care for early labor. Which question should the nurse ask? A. "What are your expectations for your current pregnancy?" B. "Have you chosen a name for the baby yet?" C. "Do you want to get up and walk around to take your mind off of the contractions?" D. "How do you like the birthing room?"

A. "What are your expectations for your current pregnancy?" A patient's psyche or emotional state can affect the response to the labor and delivery process. Assessment of emotional state is accomplished by asking the patient about previous experiences with childbirth and expectations for the current pregnancy. Asking about baby names, a desire to walk, or the birthing room reaction does not assess emotional state.

A postpartum patient delivered 30 minutes ago. Which finding should cause the nurse to closely monitor this patient? A. A soaked pad in 15 minutes B. An edematous perineum C. Patient tremors D. Fundus at the umbilicus

A. A soaked pad in 15 minutes If the perineal pad becomes soaked in a 15-minute period or if blood pools under the buttocks, continuous observation is necessary. As long as the patient remains in bed during the first hour, bleeding should not exceed saturation of one pad. An edematous perineum, tremors, and a fundus located at the umbilicus are normal postpartum findings.

The nurse is caring for a patient in labor and delivery who is 6 centimeters dilated. Which phase of labor should this finding indicate? A. Active B. Second C. Latent D. Third

A. Active This patient is in the first stage of labor, or more specifically, in the active phase, in which the cervix dilates 4 to 7 centimeters. In the latent stage of labor, the cervix dilates 0 to 3 centimeters. The patient is not in the second or third stage of labor.

The second stage of labor is prolonged and the patient is prescribed oxytocin. Which prescription should the nurse question? A. Administer 20 units intramuscularly. B. Administer 10 units in 1000 mL of intravenous fluid. C. Increase intravenous infusion 1 milliunit/min every 40 minutes. D. Administer 30 units in 500 mL of intravenous fluid.

A. Administer 20 units intramuscularly. When inducing labor, oxytocin should not be given through an intramuscular injection. The healthcare provider should be questioned about the administration route. It should be administered intravenously using an infusion pump. The other administration prescriptions are correct.

The nurse is positioning a newborn in a bassinet. For which reason should the newborn be placed in the modified Trendelenburg position? A. Aid in the drainage of mucus from the nasopharynx and trachea B. Minimize heat loss C. Auscultate the heart rate at the junction of the umbilical cord and skin D. Foster early breastfeeding and facilitate bonding

A. Aid in the drainage of mucus from the nasopharynx and trachea Because the first priority after birth is to maintain the newborn's respirations, the infant should be placed in the modified Trendelenburg position to aid drainage of mucus from the nasopharynx and trachea. The newborn's head is dried first to minimize heat loss. The heart rate at the junction of the umbilical cord and skin is auscultated as part of Apgar score evaluation. Skin-to-skin contact fosters early breastfeeding and facilitates bonding.

A patient in labor has painless vaginal bleeding. Which action should the nurse make a priority? A. Assess fetal heart rate. B. Assess fetal presentation. C. Administer oxygen by face mask. D. Place in side-lying position.

A. Assess fetal heart rate. The immediate nursing action for a patient in labor experiencing painless vaginal bleeding is to assess the fetal heart rate. Assessing fetal presentation is an immediate nursing action for a patient with greenish or brownish amniotic fluid. Administering oxygen by face mask and placing the patient in a side-lying position are immediate nursing actions if the fetal heart rate is less than 110 beats per minute

A patient in labor receives promethazine hydrochloride 50 mg. For which side effect should the nurse monitor the patient? A. Blurred vision B. Hypotension C. Nausea D. Pruritis

A. Blurred vision Blurred vision is a common side effect of promethazine hydrochloride. Hypotension is a common side effect of diphenhydramine hydrochloride. Nausea is a common side effect of several analgesics, though not promethazine hydrochloride. Pruritus is a common side effect of morphine sulfate.

The nurse is teaching an expectant couple about a partner-assisted childbirth method that focuses on natural childbirth. Which method should the nurse identify is being taught? A. Bradley B. Lamaze C. HypnoBirthing D. McRoberts

A. Bradley The Bradley method is a partner-assisted childbirth method that focuses on controlled and deep abdominopelvic breathing with a goal of achieving natural childbirth. The Lamaze method focuses on dissociative relaxation, controlled muscle relaxation, and specified breathing patterns that are used to promote birth as a normal process. HypnoBirthing focuses on breathing and relaxation techniques to help prepare the body for neuromuscular harmony, intended to make the birth process easier, safer, and more comfortable. McRoberts is a type of exercise, not a childbirth method

A couple expecting to go into labor soon asks how they can make the experience at the hospital more enjoyable. Which recommendation should the nurse make? A. Bring items from home to create a personal birthing space. B. Bring diagrams and charts of the labor process to feel prepared. C. Invite friends along to distract the woman during labor. D. Insist that the in-laws are present for the labor process.

A. Bring items from home to create a personal birthing space. The couple could bring personal items from home to help make the birthing space more enjoyable, relaxing, and personal. Bringing diagrams and charts of the birthing process, inviting friends to visit, and insisting that in-laws be present during the labor process can stress the patient and cause anxiety.

A postpartum patient with uterine atony has a history of hepatomegaly. Which prescription should the nurse question for this patient? A. Carboprost tromethamine 250 mg stat B. Carboprost tromethamine 400 mcg for severe bleeding C. Carboprost tromethamine 250 mcg every 2 hours D. Carboprost tromethamine not to exceed 12 mg

A. Carboprost tromethamine 250 mg stat In cases of immediate postpartum hemorrhage, when other measures have failed, the usual intramuscular dose of carboprost tromethamine is 250 mcg (1 mL), which can be repeated every 1.5 to 3.5 hours if uterine atony persists. In severe bleeding, an initial dose of 400 mcg may be considered. The dosage can be increased to 500 mcg (2 mL) if uterine contractility is inadequate after several doses of 250 mcg. The total dosage should not exceed 12 mg. The maximum duration of use is 48 hours.

A pregnant patient's sonogram shows the fetal head in complete extension. Which documentation should the nurse make? A. Cephalic presentation B. Complete breech C. Frank breech C. Transverse lie

A. Cephalic Presentation A fetus whose head is in complete extension is in cephalic presentation. A complete breech occurs when the fetal hips and knees are flexed. Frank breech occurs when the fetal hips are flexed and the knees are extended, with the legs against the abdomen and chest. A transverse lie occurs when the fetus has a shoulder presentation.

The nurse obtains the following data on a laboring patient: cervical dilation of 7 centimeters, transition of 5 centimeters, and mild contractions occurring every 14 minutes with a duration of 35 to 45 seconds. Which data should the nurse question? A. Cervical dilation of 7 centimeters B. Mild contraction intensity C. Contractions lasting 35 to 45 seconds D. Contractions occurring every 14 minutes

A. Cervical dilation of 7 centimeters Cervical dilatation of 7 cm indicates the active phase of labor, whereas the remaining data all signify the latent phase of labor. The cervical dilation data should be questioned, as it does not match the other findings.

The nurse is helping a patient prepare for delivery. Which action should the nurse take when cleansing this patient's perineum? A. Clean upward from the mons pubis to the lower abdomen. B. Clean downward from the lower abdomen to the mons pubis. C. Clean laterally from the right pelvic crest to the left pelvic crest. D. Clean the labia using an upward-sweeping motion.

A. Clean upward from the mons pubis to the lower abdomen After thorough handwashing, the sterile prep tray is opened, sterile gloves applied, and the vulva and perineum are cleaned with the cleansing solution. Beginning with the mons pubis, the area is cleansed up to the lower abdomen. Three sponges are used to clean the labia and vestibule with one downward sweep each.

A patient in labor is having trouble concentrating. Which psychologic disorder should the nurse expect to be documented that validates the patient's behavior? A. Clinical depression B. Obsessive-compulsive disorder C. Generalized anxiety D. Psychosis

A. Clinical depression The patient in labor exhibiting trouble concentrating may have clinical depression. Performing a ritualistic behavior to relieve an irrational impulse is associated with obsessive-compulsive disorder. Continual and excessive worrying is associated with generalized anxiety disorder. Hallucinations and delusions are associated with psychosis.

A patient in the beginning of the second stage of labor has mild respiratory acidosis. Which action should the nurse take? A. Continue to monitor the patient. B. Turn the patient on her left side. C. Rub the patient's shoulders and back. D. Administer oxygen therapy.

A. Continue to monitor the patient. As the patient enters the second stage of labor, PaCO2 levels may rise along with blood lactate levels (due to muscular activity), and mild respiratory acidosis occurs. This is a normal finding, so the patient simply requires continued monitoring. This finding does not require turning the patient on the left side or administering oxygen therapy. The patient in the second stage of labor likely does not wish to be touched.

A patient is exhibiting early decelerations on the fetal heart rate monitor. Which action should the nurse take? A. Continue to monitor. B. Provide oxygen to the patient. C. Increase intravenous fluids. D. Position the patient on the left side.

A. Continue to monitor. Early decelerations are a result of uterine contractions, causing fetal head compression and stimulating the vagus nerve. They are viewed as a reassuring pattern and the patient should be monitored. Because early decelerations are not associated with fetal hypoxia, oxygen and repositioning the patient are not appropriate actions. Since early decelerations are not associated with decreased cardiac output, increasing intravenous fluids is not an appropriate action.

The partner of a patient in early labor expresses concern that the pain of childbirth will become excruciating. Which diagnosis should the nurse add to the plan of care? A. Coping, Compromised Family B. Pain, Chronic C. Family Processes, Dysfunctional D. Breathing Pattern, Ineffective

A. Coping, Compromised Family The family during the first stage of labor is at risk for compromised coping, which can be due to anxiety, lack of knowledge, pain, and fear. The pain of childbirth is acute, not chronic. The couple is not exhibiting dysfunctional family processes. Breathing patterns in a healthy patient during labor are normal for the situation, unless an acute situation develops. (NANDA-I ©2018)

A patient in labor is requesting pain relief. Which type of analgesic should the nurse explain increases the risk of needing Pitocin to augment labor? A. Epidural B. Spinal C. Local infiltration D. Pudendal

A. Epidural An epidural increases the risk for complications, including the need for Pitocin to augment labor. Spinal, local infiltration, and pudendal analgesics do not increase the risk of needing Pitocin to augment labor.

A postpartum patient is experiencing urinary incontinence. Which type of delivery should the nurse suspect this patient experienced? A. Forceps-assisted B. Vacuum extraction C. Natural birth D. Cesarean birth

A. Forceps-assisted Urinary incontinence may occur after a forceps-assisted delivery if the urethra is damaged during the process. Urinary incontinence is not a side effect of vacuum extraction, cesarean birth, or natural birth.

A patient who is exhausted from labor has a fully dilated cervix. Which procedure should be considered for this patient? A. Forceps-assisted delivery B. Amniotomy C. Cesarean birth D. External version

A. Forceps-assisted delivery The cervix must be completely dilated for a forceps-assisted delivery to be considered as an option. An amniotomy is not needed for the patient already in labor. The cervix does not need to be completely dilated for a cesarean birth or external version.

A patient's cervix is anterior, soft, 50% effaced, and dilated at 3 cm, with the fetal head at +1 station and a Bishop score of 9. Which procedure should the nurse anticipate? A. Induction of labor B. Cesarean birth C. Forceps-assisted delivery D. Rupture of membranes

A. Induction of labor Components evaluated in a Bishop score are cervical dilatation, effacement, consistency, and position, as well as the station of the fetal presenting part. A score of 0, 1, 2, or 3 is given to each assessed characteristic. The higher the total score for all the criteria, the more likely that labor will occur. The lower the total score, the higher the failure rate. A favorable cervix is the most important criterion for a successful induction. A high Bishop score indicates that the patient is ready for labor induction. It does not indicate the need for rupture of membranes, a forceps-assisted delivery, or a cesarean birth.

A patient's fetus is in the occiput posterior position. Which symptom should the nurse anticipate? A. Intense back pain B. Vaginal bleeding C. Fetal mortality D. Cesarean birth

A. Intense back pain A patient whose fetus is in the occiput posterior position is likely to have intense low back pain. The position of the fetus would not cause vaginal bleeding. There is no risk of death to the fetus in occiput posterior position. Most fetuses in this position can be delivered vaginally, sometimes with forceps or vacuum assistance.

A patient in labor has just emptied the bladder as instructed. Which assessment should the nurse complete at this time? A. Leopold maneuvers B. Fetal heart rate C. Blood pressure D. Cervical dilation

A. Leopold maneuvers Before Leopold maneuvers are performed, the patient should empty the bladder. The bladder does not need to be emptied prior to assessment of fetal heart rate, blood pressure, or cervical dilation.

A patient in labor needs an episiotomy. Which type of anesthetic should the nurse anticipate being used? A. Local B. Spinal C. Epidural D. Pudendal

A. Local Local anesthetics are administered just before birth to provide anesthesia for an episiotomy or after birth for repair of a laceration. Spinal anesthetics are commonly used for cesarean birth. Epidurals are used to control labor pain. A pudendal block allows the use of low forceps or vacuum extraction for birth.

The nurse asks a pregnant patient to lie on her back and then proceeds to flex the patient's thighs toward the shoulders. Which childbirth exercise should the nurse explain is being demonstrated? A. McRoberts position B. Pelvic tilt C. Pelvic rock D. Kegel exercises

A. McRoberts Position The McRoberts position is performed by flexing the patient's thighs toward the shoulders while lying on the back. The other exercises do not involve positioning the patient as described.

The nurse is preparing to assess fetal heart rate with a Doppler. At which maternal location should the nurse place the diaphragm of the device? A. Midline B. Bottom of the symphysis C. Top of the umbilicus D. Left side of the abdomen

A. Midline The Doppler diaphragm should be placed on the abdomen, halfway between the umbilicus and symphysis and in the midline to assess the fetal heart rate. Fetal heart rate is not assessed by placing the Doppler at the bottom of the symphysis, top of the umbilicus, or left side of the abdomen.

A patient at 32 weeks' gestation is experiencing painless vaginal bleeding. Which action should the nurse take? A. Monitor for pain, blood loss, and uterine contractibility. B. Use an internal monitor to evaluate fetal heart rate. C. Perform a vaginal exam once per shift. D. Assess blood pressure every 4 hours

A. Monitor for pain, blood loss, and uterine contractibility. Pain, blood loss, and uterine contractibility require ongoing monitoring to assure maternal and fetal safety. Frequently assessing blood pressure is not required. Internal vaginal exams should not be performed. An external fetal monitor should be applied.

A patient in labor is prescribed an intravenous analgesic. At which time should the nurse administer the medication? A. Onset of contraction B. During contraction C. After contraction D. After delivery

A. Onset of contraction When an agent is given intravenously, the injection is best given with the onset of a contraction, when the blood flow to the uterus and the fetus is normally decreased. The medication is not given during or after a contraction. The medication is not required after delivery.

The healthcare provider is notified about a postpartum patient who just delivered the placenta. For which reason should the nurse prepare the patient for a uterine digital examination? A. Part of the placenta is missing B. Patient complaining of pain C. Abdomen is edematous D. Patient voided 500 mL clear yellow urine

A. Part of the placenta is missing Delivery of the placenta usually takes place within 30 minutes after birth. If there is a defect or a part missing from the expelled placenta, a digital uterine examination is performed. A digital uterine examination is not required for pain, abdominal bloating, or urine output.

The nurse notes that the diagnosis Coping, Compromised Family was added to the care plan for a patient in labor. For which situation should the nurse suspect this diagnosis was added? A. Partner was concerned the patient was in excruciating pain. B. Patient was asking when the baby will be born. C. Patient's mother was rubbing the patient's lower back. D. Partner was periodically walking around the delivery room.

A. Partner was concerned the patient was in excruciating pain. During the first stage of labor, the patient and family are at risk for compromised coping, which can occur due to anxiety, lack of knowledge, pain, and fear. Concern about pain would be addressed through the diagnosis of Coping, Compromised Family. Asking when the baby will be born, taking actions to help comfort the patient, and walking around the room do not indicate compromised family coping. (NANDA-I ©2018)

A pregnant patient's cervix is 3 cm dilated, -2 station with intact membranes, and fetal heart rate (FHR) of 154 beats/min. When the membranes rupture spontaneously, the FHR drops to 92 beats/min with variable decelerations. Which action should the nurse take first? A. Perform a vaginal examination. B. Administer a tocolytic as prescribed. C. Prepare for cesarean delivery. D. Notify the healthcare provider.

A. Perform a vaginal examination. Prolapsed umbilical cord can occur when the membranes rupture spontaneously. The fetus is more likely to experience variable decelerations because the amniotic fluid is insufficient to keep pressure off the umbilical cord. A vaginal exam is the best way to confirm a prolapsed cord and should be performed before notifying the healthcare provider. A tocolytic is not required and cesarean delivery is not indicated at this time.

A patient experiencing a prolonged second stage of labor is prescribed an intramuscular dose of oxytocin. Which action should the nurse take? A. Question the prescription. B. Prepare for follow-up order of misoprostol (Cytotec). C. Administer the oxytocin as prescribed. D. Set up an infusion pump instead.

A. Question the prescription When inducing labor, oxytocin should not be given through an intramuscular injection. The healthcare provider should be questioned about the administration route. Misoprostol (Cytotec) should not be given within 4 hours of oxytocin. An infusion pump should be used to administer oxytocin; however, the medication needs to be prescribed as intravenous delivery before this action can be taken.

A laboring patient is 7 centimeters dilated and restless. Which action should the nurse take? A. Reassure the patient that she won't be left alone. B. Leave the patient alone to rest. C. Turn on the patient's television. D. Encourage the patient to take an anxiolytic.

A. Reassure the patient that she won't be left alone This patient is in the transitional phase of the first stage of labor and may fear being left alone; the nurse should stay with the patient and reassure her that she will not be alone. The nurse should not turn on the patient's television unless asked to do so. The nurse should teach the patient distraction techniques instead of offering medication.

The nurse is preparing teaching materials about breastfeeding. At which time should the nurse recommend patients first try to breastfeed? A. Right after birth B. After the patient rests for at least an hour C. When the baby cries in hunger D. When the patient's prolactin is released

A. Right after birth The patient should be encouraged and helped to try breastfeeding right after birth, while the baby is awake and alert. The patient does not need to wait until after rest or until the baby cries. The act of breastfeeding will stimulate the maternal release of prolactin.

A patient in labor is diagnosed with preeclampsia. For which potential health problem should the nurse monitor the patient? A. Seizures B. Hydramnios C. Myocardial infarction D. Diabetes mellitus

A. Seizures Preeclampsia can cause several health problems in the patient in labor, including seizure activity. Preeclampsia does not cause hydramnios, myocardial infarction, or diabetes mellitus.

A newborn has an Apgar score of 7 at 1 minute. Which action should the nurse take? A. Stabilize the newborn. B. Allow the newborn to remain with the parents. C. Remove the newborn from the birthing suite. D. Permit the newborn's grandparents to visit.

A. Stabilize the newborn An Apgar score of less than 8 at 1 minute requires stabilization of the newborn. The newborn can remain with the parents, and grandparents can visit after stabilization occurs. The newborn does not need to be removed from the birthing suite.

A patient having labor induced is exhibiting hypertonicity systole uterine contractions. Which action should the nurse take? A. Stop oxytocin infusion. B. Ask the patient to bear down. C. Turn the patient to the right side. D. Prepare for imminent delivery.

A. Stop oxytocin infusion. Hypertonicity systole contractions may be caused by oxytocin. If this occurs, the infusion should be slowed or stopped and the healthcare provider notified. The patient should not push until fully dilated and effaced. Turning the patient to the right side is not an effective intervention. There is no need to prepare for delivery at this time.

A patient in labor is receiving epidural analgesia. For which reason should the nurse increase the rate of the intravenous fluid infusion? A. The patient is exhibiting signs of hypotension. B. The patient is displaying mild systemic toxicity symptoms. C. The patient is experiencing a spinal headache. D. The patient is still feeling labor pains.

A. The patient is exhibiting signs of hypotension. The patient who is exhibiting signs of hypotension should receive increased intravenous fluids to increase cardiac output and perfusion to the placenta. Mild toxicity is treated with oxygen administered by mask and a short-acting intravenous barbiturate. Spinal headaches either heal on their own or require the placement of a blood patch. The patient who is still feeling pain may need a bolus of additional medication.

A patient is experiencing anaphylactoid syndrome of pregnancy. Which outcome should be used to guide this patient's care? A. The patient is stabilized after emergency intervention. B. The patient experiences a safe vaginal delivery of the fetus. C. The patient verbalizes two causes of anaphylactoid syndrome. D. The patient verbalizes two medications used to treat anaphylactoid syndrome.

A. The patient is stabilized after emergency intervention. Anaphylactoid syndrome is life-threatening and requires emergency interventions and stabilization. The cause of anaphylactoid syndrome is unknown, so the patient would not be able to verbalize the cause. The patient experiencing anaphylactoid syndrome requires a cesarean birth. The patient verbalizing two medications used to treat the emergency is a goal, and not an outcome.

A patient in labor experiences uterine rupture. Which outcome should the nurse identify in the plan of care? A. The patient remains hemodynamically stable during cesarean delivery. B. The patient verbalizes three complications associated with hysterectomy. C. The patient is able to verbalize her blood type to ensure safe blood transfusion. D. The patient experiences vaginal delivery without hemorrhage.

A. The patient remains hemodynamically stable during cesarean delivery. Uterine rupture is a medical emergency that requires cesarean delivery. Since hemorrhage is a complication of uterine rupture, an outcome that addresses hemodynamic stability during cesarean delivery is appropriate. The patient experiencing uterine rupture cannot deliver vaginally. The patient's blood is typed and matched prior to surgery, but the patient will most likely be too distressed to answer questions about blood type. Hysterectomy is performed only if the uterus cannot be repaired or the patient is hemodynamically unstable, but verbalizing complications is a goal and not an outcome.

The nurse teaches a patient approaching her due date about Braxton Hicks contractions. Which information should the nurse include? A. They are often confused with the contractions of labor. B. They are rare in the third trimester. C. They present as a painless cramping sensation. D. They force blood into the cervix.

A. They are often confused with the contractions of labor. Braxton Hicks contractions are frequently confused with labor. These contractions are irregular and intermittent throughout all trimesters of pregnancy. Pain is typically felt in the abdomen and groin. The contractions force blood through the intervillous spaces of the placenta.

The nurse reviews the list of patients in active labor at the beginning of a scheduled shift. Which patient should the nurse monitor closely for a dysfunctional labor pattern? A. Thirty-six-year-old at 32 weeks' gestation with a macrosomic fetus B. Twenty-four-year-old at 34 weeks' gestation with ruptured membranes C. Thirty-two-year-old at 41 weeks' gestation with an estimated fetal weight of 3800 g D. Forty-four-year-old at 37 weeks' gestation with blood pressure of 154/76 mmHg

A. Thirty-six-year-old at 32 weeks' gestation with a macrosomic fetus A macrosomic fetus is a risk factor for hypotonic uterine contractions. The patients with ruptured membranes at 34 weeks' gestation and estimated fetal weight of less than 4000 g are exhibiting expected findings. Maternal hypertension does not affect labor pattern.

A patient in labor is prescribed nitrous oxide. Which instruction should the nurse provide when administering this analgesic? A. "You must insert the nasal cannula for long-term delivery." B. "Hold the mask to your face and inhale whenever you need pain relief." C. "Once you start using the nitrous oxide, you cannot take any other analgesics." D. "Nitrous oxide can sometimes increase stress and anxiety, so please report these symptoms if they develop."

B. "Hold the mask to your face and inhale whenever you need pain relief." Nitrous oxide inhalation is the only analgesia that is completely controlled by the patient. The patient holds the mask and decides when to inhale the medication and when to stop. A nasal cannula is not used. The use of nitrous oxide does not prevent the patient from moving to a different form of analgesia, including an epidural regional block. Nitrous oxide has anxiolytic properties.

A patient in the first stage of labor asks about the role of the nurse during childbirth. Which response should the nurse make? A. "I will be providing the physician assistance while the nursing assistant monitors your vital signs." B. "I will be assessing you regularly and more frequently as labor progresses." C. "I will bring you any requested food or beverages while you're in labor." D. "I will be available for support but will only assess you if there's a sign that something is going wrong."

B. "I will be assessing you regularly and more frequently as labor progresses." Labor support is a primary nursing role during the first stage of labor. The patient and family look to their nurse for information about labor progress, pain management, procedures, and examinations, and for assistance with pain management techniques. The nurse monitors all vital signs. The patient will not be eating and drinking during labor. The nurse will provide reassurance that labor is progressing and explain any deviations from normal, should they occur.

The nurse is evaluating a postpartum patient in the fourth stage of labor. Which patient statement should demonstrate achievement of an outcome? A. "I know I will need to deliver the placenta soon." B. "I'm making sure my baby is warm enough, but not too hot." C. "I understand the importance of the Apgar score." D. "I hope my baby is normal and healthy."

B. "I'm making sure my baby is warm enough, but not too hot." An appropriate outcome for a patient in the fourth stage of labor is supporting the physical well-being of the newborn. The patient anticipates delivery of the placenta and the newborn's Apgar score is assessed in the third stage of labor. The patient in the fourth stage of labor should not worry about the baby's health but instead be actively engaged in protecting it.

The nurse is teaching a pregnant patient about a forceps-assisted delivery. Which patient statement should indicate that further teaching is required? A. "If my baby's skull is below +2 station, low forceps will be used." B. "If my baby is above the pelvic inlet, high forceps will be used." C. "If my baby's skull is above +2 station, midforceps will be used." D. "If my perineum is bulging, outlet forceps will be used."

B. "If my baby is above the pelvic inlet, high forceps will be used." High forceps are not indicated in current obstetric practice, and the patient whose fetus is at a station of -3 or above the pelvic inlet should give birth by cesarean. The other statements regarding the use of midforceps, low forceps, and outlet forceps are accurate.

The nurse is guiding a laboring patient through visualization and imagery. Which statement should the nurse make? A. "Imagine your baby being born with no defects." B. "Imagine your favorite vacation spot." C. "Imagine your baby coming into the world silently." D. "Imagine taking the subway home with your new baby."

B. "Imagine your favorite vacation spot." Asking the patient to visualize a favorite place can help reduce stress and anxiety during labor and delivery. Asking the woman to visualize her baby being born with no defects can induce stress in the woman. Asking the woman to visualize her baby being born silently is anxiety-provoking, as the lack of crying could mean the baby is in distress. Asking the woman to visualize holding her baby on a dirty, crowded subway does not generate feelings of peacefulness.

An expectant couple is learning about pain relief options during labor. Which statement should indicate teaching was effective? A. "If a cesarean delivery is required, a pudendal block may be used." B. "Local anesthetics provide pain relief for an episiotomy or laceration repair." C. "A spinal block will likely be used if forceps are needed." D. "Epidural analgesia must be administered continuously to be effective."

B. "Local anesthetics provide pain relief for an episiotomy or laceration repair." Local anesthetics are administered just before birth to provide anesthesia for an episiotomy or after birth for repair of a laceration. A spinal block is commonly used for cesarean birth. A pudendal block allows the use of low forceps or vacuum extraction for birth. Epidurals can be given either as a bolus or as a continuous infusion.

The nurse is teaching a pregnant patient whose fetus has an estimated weight of 4300 g. Which statement should indicate that teaching was effective? A. "I can still have a vaginal birth with low risk." B. "My baby could have damage to the nerves in the shoulder, arm, and hand." C. "My baby's blood sugar will be high during delivery." D. "Cesarean birth is my only option."

B. "My baby could have damage to the nerves in the shoulder, arm, and hand." A fetus that weighs 4000 g or more is considered macrosomic. Shoulder dystocia is the most significant complication in macrosomia and can cause injury to the nerves in the shoulder, arm, and hand due to improper or excessive traction applied to the fetal head during birth. Vaginal birth of the macrosomic fetus increases the risk of perineal lacerations, episiotomy, postpartum hemorrhage, and puerperal infection. A cesarean birth may be required for a fetus greater than 4500 g, but the best method of birth for a fetus weighing 4000 to 4500 g is debated.

A patient in labor who included the use of pain medication in the birth plan asks why it was given at a specific time without it being requested. Which response should the nurse make? A. "The healthcare provider prescribed a dose for you at that time." B. "Pain medication during labor is most effective when timed with my assessment findings." C. "You are unable to feel the pain, so I knew you wouldn't request it." D. "All patients in labor receive pain medication at this time."

B. "Pain medication during labor is most effective when timed with my assessment findings." In many organizations, the nurse does not give the analgesic agent when prescribed by the healthcare provider, but rather when the nurse decides the timing is right. The patient is likely experiencing pain. Medication for pain is based upon the patient's needs.

The nurse is teaching the parents of a newborn who was delivered with a brow presentation. Which statement should indicate that teaching was effective? A. "This presentation occurs frequently in patients who have had more than one pregnancy." B. "The baby's facial swelling will go away with time." C. "I am at a higher risk for postpartum hemorrhage." D. "The baby is at high risk for a neuromuscular disorder."

B. "The baby's facial swelling will go away with time." Any facial edema and head molding that result from delivery of a fetus who had a brow presentation will subside in a few days. The fetus delivered in breech presentation is more likely to have neuromuscular delays. The patient with a macrosomic fetus is at higher risk of postpartum hemorrhage. Face presentation occurs frequently in multiparous patients, and brow presentation is the least common of all presentations.

A patient in the second stage of labor reports an uncontrollable urge to bear down, but fears being torn apart. Which response should the nurse make? A. "I'll ask the healthcare provider to check your perineum right away." B. "This is normal. Just breathe and push at the start of the next contraction." C. "Tightening your muscles will relieve the pressure." D. "Stay calm and remain in the supine position until I give you the signal to push."

B. "This is normal. Just breathe and push at the start of the next contraction." In the second stage of labor, the patient will feel an uncontrollable urge to bear down, and may also fear that damage is being caused due to the pain. The patient should be reassured that these sensations are normal and should be encouraged to breathe and push when the next contraction begins. Because this is normal, the healthcare provider does not need to check the perineum. The patient should bear down rather than tightening the muscles. An upright position or lateral positon is preferred over supine positioning, and the patient who feels the urge to push should not wait for a signal.

A patient in labor is admitted to the birthing unit. Which question should the nurse ask to learn the patient's birth plan? A. "Will your mother be arriving soon?" B. "Who will serve as your labor partner?" C. "What will be the baby's name?" D. "Will anyone be helping you at home after discharge?"

B. "Who will serve as your labor partner?" After admission to the birthing unit, the patient's birthing plan and expectations for the delivery process should be acknowledged. Asking who will be in the room during labor clarifies the patient's plans. It should not be assumed that the patient's mother will be part of the birthing process. The baby's name and what will happen after discharge are not necessary to know at this time.

The nurse is monitoring a low-risk patient who is in labor. Which action should the nurse take first? A. Insert internal pressure catheter. B. Assess fetal heart tones. C. Apply fetal scalp electrode. D. Prepare patient for telemetry.

B. Assess fetal heart tones. A low-risk patient in labor should be assessed with the least invasive method before moving to a more invasive method, if indicated. Assessing fetal heart tones allows the nurse to hear the fetus's heart sounds without implementing an invasive technique.

A postpartum patient received a continuous epidural from the first stage of labor until after delivery. For which health problem should the nurse assess the patient? A. Nausea B. Bladder distention C. Vomiting D. Uterine atony

B. Bladder distention An epidural block can cause bladder distention. Nausea, vomiting, and uterine atony are not caused by epidural anesthesia.

A patient has just given birth. Which activity should the nurse encourage? A. Showering B. Breastfeeding C. Cleansing of the perineum D. Bowel movement

B. Breastfeeding The patient should be encouraged and helped to try breastfeeding right after birth, while the baby is awake and alert. The patient can shower after birth if desired, but this should be left up to the patient. The nurse cleanses the perineum. The patient should empty the bladder after birth.

A newborn has temporary facial swelling. Which fetal malpresentation should the nurse expect to see documented in the patient's medical record? A. Transverse lie B. Brow presentation C. Breech presentation D. Compound presentation

B. Brow presentation Any facial edema and head molding that result from delivery of a baby in brow presentation will subside in a few days. Facial swelling is not associated with transverse lie, breech presentation, or compound presentation.

A postpartum patient is being prepared for delivery of the placenta. For which reason should oxytocin 20 units be given to the patient at this time? A. Increase breast milk production B. Contract the uterus and minimize bleeding C. Decrease maternal blood pressure D. Relax the perineal muscles

B. Contract the uterus and minimize bleeding Oxytocin is given to contract the uterus and minimize bleeding. Oxytocin does not have an effect on breast milk production or maternal blood pressure and is not used to relax perineal muscles.

The chorionic villi of a pregnant patient's placenta have penetrated the myometrium of the uterus. Which equipment should the nurse have readily available? A. Fetoscope B. Crash cart C. Blood glucose monitor D. Sequential compression device

B. Crash cart Placenta percreta is the penetration of the placental chorionic villi into the myometrium of the uterus. This is a life-threatening situation that may require the use of a crash cart if the patient experiences cardiac arrest. The sequential compression device, blood glucose monitor, and fetoscope are not indicated in the care of a patient experiencing placenta percreta.

The nurse reviews the history of a patient in labor who was admitted to the birthing unit. Which finding should the nurse immediately report to the healthcare provider? A. Intimate partner violence B. Dark red vaginal bleeding in the third trimester C. Difficulty breastfeeding in previous pregnancies D. Fatigue

B. Dark red vaginal bleeding in the third trimester Dark red bleeding in the third trimester can indicate a placenta previa that should be reported to the healthcare provider. The patient experiencing intimate partner violence is at risk for harm after delivery, but this is not the priority at this time. Difficulties with breastfeeding can be addressed after the birth of the baby. Fatigue is a normal finding during pregnancy.

A patient at 42 weeks of gestation is being prepared for induction of labor. Which information should the nurse expect to find in the patient's medical record? A. Rupture of amniotic fluids B. Decreased amniotic fluid volume measured via ultrasound C. Increased amniotic fluid volume performed via amnioinfusion D. Scheduled cesarean birth

B. Decreased amniotic fluid volume measured via ultrasound In the event of a postterm pregnancy, induction of labor is recommended when an ultrasound indicates a decrease in amniotic fluid. Amnioinfusion is performed to increase the volume of fluid during labor, not before induction. Cesarean birth is not indicated for this health problem. The patient whose membranes have ruptured does not need to be induced.

A patient who has just given birth expels a placenta that is missing a portion. Which action should the nurse anticipate? A. Clamping of the umbilical cord B. Digital uterine examination C. Application of nasal oxygen at 3 liters D. Administration of more oxytocin

B. Digital uterine examination If there is a defect or a part missing from the expelled placenta, a digital uterine examination is performed. Clamping and cutting of the umbilical cord occurs before expulsion of the placenta. The patient does not require oxygen. More oxytocin is not needed because the placenta has already been delivered.

The nurse is caring for a patient who is in active labor. Which finding should support this conclusion? A. Dilated 0 to 3 centimeters B. Dilated 4 to 7 centimeters C. Dilated 8 to 10 centimeters D. Dilated 11 to 15 centimeters

B. Dilated 4 to 7 centimeters During the active phase of labor, the cervix dilates 4 to 7 centimeters. In the latent stage of labor, the cervix dilates 0 to 3 centimeters. In the transition stage of labor, the cervix dilates 8 to 10 centimeters. Dilation of 11 to 14 centimeters usually does not occur and is not assigned a phase of labor.

A patient in labor requires an episiotomy. Which should the nurse explain as a potential complication of this procedure? A. Urinary incontinence B. Dyspareunia C. Prolonged hospital stay D. Hematomas

B. Dyspareunia An episiotomy can result in perineal discomfort that may continue for days or weeks past birth, including dyspareunia, or painful intercourse. Urinary incontinence, hematomas, and prolonged hospital stay are not complications of an episiotomy.

The fetus of a patient in the second stage of labor has descended 0.7 cm over the past hour. Which finding should the nurse document? A. Preterm labor B. Dystocia C. Hypotonic labor D. Postterm labor

B. Dystocia Dystocia is defined as a rate of cervical dilation less than 0.5 cm per hour over 4 hours during the active stage of labor, or less than 1 cm per hour of fetal descent during the second stage of labor. This patient is not exhibiting hypotonic labor, postterm labor, or preterm labor.

The nurse monitors the fetal heart monitor of a patient in labor. For which reason should the nurse provide no further action at this time? A. Recurrent late decelerations B. Early decelerations C. Sinusoidal patterns D. Bradycardia

B. Early decelerations Early decelerations are a result of uterine contractions, causing fetal head compression and stimulating the vagus nerve. They are viewed as a reassuring pattern and monitoring should continue. Recurrent late decelerations, bradycardia, and sinusoidal fetal heart rate patterns all require immediate intervention.

The nurse is preparing to assess a patient in labor. Which action should the patient complete first in preparation for Leopold maneuvers? A. Place a pillow behind the shoulders. B. Empty the bladder. C. Bend the knees. D. Declare when a contraction begins.

B. Empty the bladder. Before Leopold maneuvers are performed, the patient should empty the bladder. After this, a pillow can be placed behind the shoulders and the knees bent. Leopold maneuvers should be completed between contractions.

A multiparous patient with a pendulous abdomen from obesity is in labor. Which fetal presentation should the nurse anticipate? A. Brow B. Face C. Breech D. Mentum

B. Face Face presentation occurs frequently in multiparous patients and patients with a pendulous abdomen. The multiparous patient or patient with a pendulous abdomen is not more inclined to a brow, breech, or mentum presentation.

The nurse is reviewing care for a patient in labor who is taking nitrous oxide for pain relief. Which nursing diagnosis should be the priority? A. Pain, Labor B. Falls, Risk for C. Toileting Self-Care Deficit D. Sleep Pattern, Disturbed

B. Falls, Risk for The patient receiving analgesics and anesthetics during labor is at risk for falls. Although the patient may have labor pains, trouble toileting, and a disturbed sleep pattern, safety is always the priority nursing diagnosis. (NANDA-I ©2018)

The nurse is caring for a patient with abruptio placentae. Which nursing diagnosis should be included in the patient's care plan? A. Tissue Integrity, Risk for Impaired B. Fluid Volume, Risk for Deficient C. Self-Concept, Readiness for Enhanced D. Self-Neglect

B. Fluid Volume, Risk for Deficient The patient with abruptio placentae is at risk for hemorrhage, which also puts the patient at risk for fluid volume deficiency. There is no indication the patient is self-neglectful or in need of enhanced self-concept. The patient is at risk for tissue perfusion, not impaired integrity. (NANDA-I ©2018)

The nurse is caring for a patient in the second stage of labor who is in obvious pain. For which reason should the nurse understand the patient is in pain? A. Retention of the vagina and perineum B. Hypoxia of contracting uterine muscles C. Stretching of the cervix D. Pressure on the chest and upper back

B. Hypoxia of contracting uterine muscles During the second stage of labor, pain is due to hypoxia of the contracting uterine muscle cells, distention of the vagina and perineum, and pressure on adjacent structures, including the lower back, buttocks, and thighs. The primary source of pain in the first stage of labor is stretching of the cervix.

The nurse is preparing teaching materials on the stages of labor. At which time should the nurse note the fourth stage of labor begins? A. When the patient begins pushing B. Immediately after expulsion of the placenta C. Once the newborn is stable D. After the patient's vital signs return to baseline

B. Immediately after expulsion of the placenta The period immediately following expulsion of the placenta is referred to as the fourth stage of labor and birth. The patient pushes in the second stage of labor. The newborn is stabilized in the third stage of labor. The end of the fourth stage of labor occurs when the patient's vital signs return to baseline.

A patient is in labor. For which reason should the nurse assist the patient into the left lateral Sims position? A. Allows gravity to aid in the descent and expulsion of the newborn. B. Increases perineal relaxation and decreases the need for an episiotomy. C. Increases the size of the pelvic outlet. D. Increases placental and umbilical blood flow and decreases nonreassuring fetal status.

B. Increases perineal relaxation and decreases the need for an episiotomy. The left lateral Sims position increases perineal relaxation and decreases the need for an episiotomy. Sitting and squatting positions allow gravity to aid in the descent and expulsion of the baby. Squatting increases the size of the pelvic outlet. The hands and knees position increases placental and umbilical blood flow and decreases nonreassuring fetal status.

The nurse teaches an expectant couple about childbirth methods. Which information should be provided regarding the Bradley method? A. It is a psychoprophylactic method that focuses on breathing patterns. B. It is a partner-assisted method that focuses on natural childbirth. C. It is a method that focuses on neuromuscular harmony. D. It is a method that focuses on the woman's musculature.

B. It is a partner-assisted method that focuses on natural childbirth. The Bradley method is a partner-assisted childbirth method that focuses on controlled and deep abdominopelvic breathing with a goal of achieving natural childbirth. The Lamaze method focuses on dissociative relaxation, controlled muscle relaxation, and specified breathing patterns to promote birth as a normal process. HypnoBirthing focuses on breathing and relaxation techniques to help prepare the body for work in neuromuscular harmony with the intent to make the birth process easier, safer, and more comfortable. There is no childbirth method that focuses on the woman's musculature.

The nurse prepares a patient for a transversus abdominis plane (TAP) block. Which should the nurse explain is an advantage of this type of pain management? A. Enhances effects of opioids B. Lasts for 12 hours C. Requires only one injection D. Has no contraindications

B. Lasts for 12 hours TAP blocks can provide up to 12 hours of postoperative analgesia without the need for additional opioids, reducing or even eliminating the need for intravenous opioids for patients who required general anesthesia for surgery. TAP blocks reduce opioid-related side effects such as sedation, nausea and vomiting, and decreased bowel motility. A TAP block is a low-risk procedure that involves two injections into the fascia of the abdominal wall. The only contraindications for the procedure are infection at the injection site, refusal or inability to consent by the patient, or allergies to local anesthetics.

The nurse is observing a patient's electronic fetal monitor. Which finding should require immediate intervention? A. Early decelerations with contractions B. Late decelerations with minimal variability C. Variable decelerations that recover to baseline D. Accelerations

B. Late decelerations with minimal variability Late decelerations are considered a nonreassuring fetal heart rate (FHR) pattern, and therefore require immediate intervention. Early decelerations are usually benign. Variable decelerations indicate cord compression, but those that recover to the baseline indicate that the fetus is tolerating the decelerations. Accelerations of the fetal heart rate indicate good oxygen reserve.

The nurse is assessing fetal heart rate in a pregnant patient. Which part of the abdomen should be used if the fetus is in cephalic presentation? A. Above the umbilicus B. Lower quadrant C. Below the umbilicus D. Upper quadrant

B. Lower quadrant When the fetus is in a cephalic presentation, the fetal heart rate (FHR) is best heard in the lower quadrant of the maternal abdomen. In a breech presentation, the FHR is heard at or above the maternal umbilicus. In a transverse lie position, the FHR may be heard best just above or just below the maternal umbilicus. Fetal heart rate is never assessed in the upper maternal abdomen

A patient in labor is prescribed nalbuphine hydrochloride (Nubain). Which additional medication should the nurse have available when giving this narcotic to the patient? A. Acetaminophen (Tylenol) B. Naloxone (Narcan) C. Promethazine (Phenergan) D. Diphenhydramine (Benadryl)

B. Naloxone (Narcan) The nurse should prepare for a precipitous birth after nalbuphine administration by having naloxone (Narcan) nearby, due to the increased risk of neonatal resuscitation. The other medications do not reverse respiratory depression.

A pregnant patient wants an epidural during labor. Which potential complication should the nurse explain to this patient? A. Trauma to the sciatic nerve B. Need for oxytocin (Pitocin) to augment labor C. Hypertension D. Broad ligament hematoma

B. Need for oxytocin (Pitocin) to augment labor An epidural increases the risk for complications, including the use of Pitocin to augment labor. Hypotension is a complication of an epidural. Trauma to the sciatic nerve and broad ligament hematoma are complications of a pudendal block.

The fetus of a patient in the first stage of labor is in the occiput posterior position. In which way should the nurse position the patient? A. Standing B. On hands and knees C. Squatting D. Sitting

B. On hands and knees When the fetus is in the persistent occiput posterior position, side-lying or hands and knees positions can promote fetal head rotation. Standing, squatting, and sitting in an upright position may all be beneficial during labor, but are not the best positions for a patient whose fetus is in the occiput posterior position.

The nurse receives handoff communication from the previous nurse. Which patient should the nurse assess first? A. Patient at 32 weeks' gestation whose fetal monitor strip shows late decelerations B. Patient at 28 weeks' gestation who finds blood in the toilet after urination C. Patient at 37 weeks' gestation whose membranes ruptured spontaneously D. Patient at 35 weeks' gestation who has a strong urge to push

B. Patient at 28 weeks' gestation who finds blood in the toilet after urination Painless, bright red vaginal bleeding is the best diagnostic sign of placenta previa. Because placenta previa can be potentially lethal to the patient and fetus, the patient with blood in the toilet after urination is the highest priority. Late fetal decelerations are an abnormal finding, but this is not the priority patient. The patient with spontaneous rupture of membranes and the patient with an urge to push are not at risk.

The fetus of a patient in preterm labor is exhibiting signs of stress and requires perfusion. Which action should the nurse take? A. Elevate the head of the bed 45 degrees. B. Position the patient on the left side. C. Place the patient in the prone position. D. Place the patient in Trendelenburg position.

B. Position the patient on the left side. When the patient and fetus are exhibiting signs of stress, repositioning the patient on the left side increases perfusion to both the fetus and patient. Placing a pregnant patient in Trendelenburg is usually reserved for a prolapsed umbilical cord. The prone position does not increase perfusion. Elevating the head of the bed 45 degrees can increase the maternal blood pressure and decrease perfusion to the fetus.

A pregnant patient receives a prostaglandin vaginal suppository to promote ripening of the cervix. Which action should the nurse take next? A. Shave the patient's perineum. B. Position the patient supine. C. Insert an indwelling catheter. D. Apply an internal fetal monitor.

B. Position the patient supine. After insertion of a prostaglandin, the patient should be supine for 2 hours. There is no reason to insert an indwelling catheter. An internal fetal monitor cannot be applied until adequate cervical dilatation has occurred. The perineum does not need to be shaved at this time.

A pregnant patient is concerned that the head of the fetus is on one side of the abdomen and the breech is on the opposite side of the abdomen. Which diagnosis should the nurse include in the patient's plan of care? A. Activity Intolerance, Risk for B. Powerlessness, Risk for C. Decision-Making, Readiness for Enhanced D. Perioperative Positioning Injury, Risk for

B. Powerlessness, Risk for The patient's fetus is in transverse lie presentation, in which the fetus is lying horizontally across the patient's abdomen. The patient may feel powerless in this situation as the birth outcome is unknown. The patient is not at risk for activity intolerance or perioperative positioning injury. There is no indication that the patient's decision-making is impaired. (NANDA-I ©2018)

A nulliparous patient who has experienced no change in cervical dilation for 2 hours is diagnosed with cephalopelvic disproportion. Which action should the nurse take? A. Assess the position and presentation of the fetus. B. Prepare for cesarean birth. C. Prepare for amniotomy. D. Administer oxytocin (Pitocin).

B. Prepare for cesarean birth. If labor is not progressing, the patient should be assessed for cephalopelvic disproportion (CPD). If CPD exists, a cesarean birth is indicated. Oxytocin augmentation and amniotomy should only be used if CPD does not exist. The position and presentation of the fetus would be assessed before diagnosing CPD.

A patient at 28 weeks' gestation has uterine contractions every 5 minutes for 20 minutes. For which health problem should the nurse initiate care? A. False labor B. Preterm labor C. Premature labor D. Postterm labor

B. Preterm labor Preterm labor is defined as uterine contractions every 5 minutes for 20 minutes, or 8 contractions in a 60-minute period, in a patient between 20 and 36 6/7 weeks' gestation. The patient's contractions do not indicate postterm or false labor. There is no such thing as premature labor.

A pregnant patient is attempting a vaginal birth after cesarean. Which finding in the patient's medical record should indicate the highest success rate? A. Patient was diagnosed with dystocia in the second stage of labor. B. Previous cesarean was performed for breech presentation. C. Patient was diagnosed with dystocia before 5-cm dilatation. D. Previous cesarean was performed for nonreassuring fetal status.

B. Previous cesarean was performed for breech presentation. Patients whose previous cesarean was performed for breech presentation have the highest success rates for vaginal birth after cesarean. Cesarean performed for nonreassuring fetal status, dystocia before 5-cm dilatation, and dystocia in the second stage have a lower success rate.

The nurse observes a fetus whose acromion process of the scapula is toward the woman's right, and the fetus's back is posterior. Which documentation should the nurse make for this presentation? A. RADA B. RADP C. RSA D. RST

B. RADP The right-acromion-dorsal-posterior presentation is documented as RADP. RADA is the right-acromion-dorsal-anterior presentation. RSA is the right-sacrum-anterior presentation. RST is the right-sacrum-transverse presentation.

A patient gives birth via vacuum extraction. For which health problem should the nurse assess the newborn? A. Macrosomia B. Scalp lacerations and bruising C. Edema D. Caput succedaneum

B. Scalp lacerations and bruising Neonatal complications of vacuum extraction include scalp lacerations and bruising. Edema is a maternal complication of vacuum extraction. Caput succedaneum is a neonatal complication of a forceps-assisted delivery. Fetal macrosomia is a contraindication to the use of vacuum extraction, and not a complication.

A patient is progressing through labor. During which stage of labor should the nurse continuously palpate the patient's contractions? A. First B. Second C. Active D. Transition

B. Second During the second stage of labor, the patient's contractions will be continuously palpated. The first stage of labor includes the active and transition phases, during which the patient's contractions will be palpated every 15 to 30 minutes.

A pregnant patient of Native American heritage has a history of periodontal disease. For which potential health problem should the nurse plan care for this patient? A. Fetal malpresentation B. Spontaneous preterm labor C. Large for gestational age fetus D. Placenta previa

B. Spontaneous preterm labor Risk factors for spontaneous preterm labor include Native American ancestry and periodontal disease. This patient is not at any higher risk for placenta previa, fetal malpresentation, or large for gestational age fetus.

The nurse is assessing the fetal heart rate in a patient with polyhydramnios. Which outcome should the nurse expect? A. The fetal heartbeat is absent. B. The fetal heartbeat is difficult to hear. C. The fetal heartbeat indicates tachycardia. D. The fetal heartbeat is irregular.

B. The fetal heartbeat is difficult to hear It may be difficult to hear the fetal heartbeat in a patient with polyhydramnios due to the extra fluid. Polyhydramnios does not cause an absent heartbeat, tachycardia, or an irregular heartbeat.

A pregnant patient is diagnosed with a hemorrhage. Which outcome should the nurse include in the plan of care? A. The fetus's oxygen saturation is maintained during labor and delivery. B. The patient verbalizes an understanding of the reasons for medical therapy. C. The patient does not require analgesia during labor and delivery. D. The patient verbalizes three reasons for medical therapy before the first stage of labor.

B. The patient verbalizes an understanding of the reasons for medical therapy. It is important for the patient to verbalize the reasons for medical therapy. The patient verbalizing three reasons for medical therapy before the first stage of labor is an appropriate goal, not an outcome. An outcome that solely focuses on the fetus does not account for the patient's care. A patient will need analgesia during labor and delivery.

A pregnant patient asks about the benefit of childbirth preparation classes. Which advantage should the nurse describe? A. They recommend the birth setting most appropriate for each patient. B. They teach relaxation and coping techniques. C. They help a patient lengthen the time in labor for a more natural process. D. They prepare the patient for labor and birth, but do not provide any benefit to the partner.

B. They teach relaxation and coping techniques. Childbirth preparation classes vary in coverage of subjects related to the maternity cycle, but all teach relaxation and coping techniques, as well as what to expect during labor and birth. They do not recommend a specific birth setting, but rather help patients understand the choices available. Childbirth methods taught in these classes have been shown to shorten labor. The classes can benefit both parents, for whom childbirth becomes a shared and profound emotional experience.

A pregnant patient experiences strong contractions every 2 minutes for 90 seconds and changes positions frequently. Which phase of labor should the nurse document for this patient? A. Active B. Transition C. Latent D. Second

B. Transition During the transition phase of labor, contractions occur every 1.5 to 2 minutes for a duration of 60 to 90 seconds. The contractions are strong in intensity and the patient may demonstrate significant anxiety, such as changing positions frequently. In the active phase of labor, contractions begin to increase in intensity, occurring every 2 to 5 minutes for a duration of 40 to 60 seconds. The latent phase of labor is characterized by mild contractions that occur every 3 to 30 minutes for a duration of 20 to 40 seconds. There is no second phase of labor, and the patient in the transition phase of labor is in the first stage.

A patient in the second trimester of pregnancy is experiencing spotting. For which diagnostic test should the nurse prepare the patient? A. Nonstress test B. Ultrasound C. Vaginal examination D. Contraction stress test

B. Ultrasound Spotting in the second trimester can indicate placenta previa, which is diagnosed through the use of a transabdominal ultrasound scan. A nonstress test or contraction stress test would not be used to diagnose placenta previa. Indirect diagnosis is made by localizing the placenta via tests that require no vaginal examination.

The electronic monitor shows repetitive variable decelerations in the fetal heart rate of a patient in labor. Which health problem should the nurse anticipate? A. Breech presentation B. Umbilical cord compression C. Fetal transverse lie D. Fetal hypoglycemia

B. Umbilical cord compression Fetal variable decelerations occur when there is umbilical cord compression. Breech presentation alone does not cause variable decelerations. Fetal transverse lie and fetal hypoglycemia do not cause variable decelerations.

A patient in labor admits to using cocaine. Which situation should the nurse anticipate? A. Prolonged labor B. Uterine tachysystole C. Placenta previa D. Uterine anamoly

B. Uterine tachysystole Maternal cocaine use is a risk factor for tachysystole, which is defined as uterine contraction frequency of more than five contractions in a 10-minute period with less than 60 seconds of relaxation between contractions, or contractions lasting greater than 2 minutes. Cocaine use does not increase the risk of prolonged labor, placenta previa, or uterine anamoly.

A patient whose fetus is in breech position requests to use moxibustion since it was recommended by a holistic healthcare provider. Which response should the nurse make? A. "Moxibustion causes you to inhale smoke, which is bad for the baby." B. "We don't allow smoking of any kind in this facility." C. "I'll ask the certified nurse midwife if it's permitted." D. "You are not Chinese, so I'm not sure why you would request this."

C. "I'll ask the certified nurse midwife if it's permitted." Moxibustion is a traditional Chinese medicine (TCM) that, when used in pregnancy to increase fetal activity, involves burning mugwort on the outer corner of the fifth toenail. The healthcare provider should be consulted before permitting the patient to engage in moxibustion. Moxibustion does not produce enough smoke to harm the patient or fetus. The patient should be permitted to explore alternative therapies regardless of heritage.

A patient in labor asks for a "pain pill." Which response should the nurse make? A. "It is not safe for you to swallow anything during labor." B. "There are no analgesics that can be taken orally." C. "Oral analgesics take too long to work during labor." D. "If that would be easier for you, I can ask the healthcare provider."

C. "Oral analgesics take too long to work during labor." Oral analgesics are not used because they are poorly absorbed and gastric emptying time is prolonged during labor. It is generally safe to swallow during labor in between contractions. There are oral analgesics, but they are not used during labor. The healthcare provider does not need to be contacted.

A pregnant patient who has been identified as having an anthropoid pelvis asks what this means. Which statement should the nurse make? A. "Labor will be more difficult for you." B. "You may require a cesarean birth." C. "You will have an easier vaginal childbirth." D. "This has no effect on childbirth."

C. "You will have an easier vaginal childbirth." The diameters of an anthropoid pelvis are larger and thus indicate an easier vaginal childbirth. A patient with an android or platypelloid pelvis would find labor more difficult and could require a cesarean birth. Pelvis type does affect labor and childbirth.

The nurse reviews information received during handoff communication. Which patient should the nurse identify as having the greatest risk for a prolapsed umbilical cord? A. 39 weeks' gestation, 9 cm dilated, 100% effaced, +1 station, ruptured membranes B. 40 weeks' gestation, 5 cm dilated, 100% effaced, -2 station, intact membranes C. 40 weeks' gestation, 5 cm dilated, 100% effaced, -2 station, ruptured membranes D. 38 weeks' gestation, 3 cm dilated, 80% effaced, 0 station, intact membranes

C. 40 weeks' gestation, 5 cm dilated, 100% effaced, -2 station, ruptured membranes Prolapse is more likely to occur in a patient with an abnormal fetal presentation, such as high station. Rupture of amniotic membranes also increases the risk of cord prolapse. The chance of a prolapsed cord is less if the presenting fetal part is engaged.

A patient who is 33 weeks pregnant with twins is expected to deliver in the next 5 days. Which intervention should the nurse perform? A. Prepare for a blood transfusion. B. Administer magnesium sulfate as prescribed. C. Administer corticosteroids as prescribed. D. Send the placenta to the laboratory.

C. Administer corticosteroids as prescribed. The administration of corticosteroids is now recommended for multiple pregnancies between 24 and 34 weeks if a risk to deliver within the next 7 days is suspected. Preparing for a blood transfusion is not necessary at this time. Magnesium sulfate should only be administered if the fetus is preterm. Sending the placenta to pathology to determine if same sex twins are monozygotic or dizygotic occurs after delivery, if indicated.

A patient asks how the baby's condition is determined during labor. Which information should the nurse provide? A. A pelvic exam will be performed every 2 hours. B. Blood serum diagnostics are performed every shift. C. An electronic fetal monitor is used to assess the fetus's heart rate D. Pain is assessed in the patient's abdomen.

C. An electronic fetal monitor is used to assess the fetus's heart rate The fetal heart rate is an indicator of how the fetus is progressing. Pelvic exams are performed during labor, but these do not provide information about the status of the fetus. Blood serum diagnostics are not completed during labor. Pain during labor is expected and is not a way to determine the status of the fetus.

A patient is in the second stage of labor. Which action should the nurse take to promote patient comfort? A. Place icepacks on the vulva. B. Continuously massage the perineum. C. Apply a cool washcloth to the forehead. D. Lay a blanket over the abdomen.

C. Apply a cool washcloth to the forehead. Cool cloths to the face and forehead may help provide cooling as the patient is involved in the intense physical exertion of pushing. Placing icepacks on the vulva interferes with the delivery. Perineal massage, if used, should be limited and not continuous, as it can lead to perineal edema. Covers are typically removed as the patient feels hot during labor.

The fetus of a patient in labor is displaying shoulder dystocia. Which action should the nurse take? A. Prepare for immediate cesarean birth. B. Prepare to administer two units of packed red blood cells. C. Assist the patient into McRoberts maneuver. D. Position the patient in the left side-lying position.

C. Assist the patient into McRoberts maneuver. McRoberts maneuver is used to change the maternal pelvic angle and reduce the force needed to extract the fetal shoulders. This will decrease the incidence of brachial plexus stretching and clavicular fracture. The patient does not require an emergency cesarean birth or blood transfusion. The patient would be positioned with the thighs up against the abdomen.

A pregnant patient has the amniotic membranes stripped. For which health problem should the nurse monitor the patient? A. Cervical contractions B. Low back pain C. Bloody discharge D. Hypotension

C. Bloody discharge Stripping of the membranes can cause significant discomfort and the side effects of uterine contractions, cramping, scant bleeding, and a bloody discharge. Stripping does not cause low back pain, cervical contractions, or hypotension.

A pregnant patient with a history of multiple gestation is diagnosed with placenta previa. Which fetal presentation should the nurse expect to document? A. Occiput B. Brow C. Breech D. Face

C. Breech Breech presentation is most frequently associated with placenta previa. Face, brow, and occiput presentations are not strongly associated with placenta previa.

A multiparous patient with a congenitally short cervix arrives for a prenatal visit. Which procedure should the nurse expect to find in the patient's history? A. Dilatation and curettage (D and C) B. Hysterectomy C. Cerclage D. Elective abortion

C. Cerclage A multiparous patient with a congenitally short cervix most likely has a past history of cerclage to help maintain the fetus in utero until time for delivery. A congenitally short cervix would not be linked with elective abortion, hysterectomy, or D and C.

The nurse is guiding a patient through labor. At which point should the nurse ask the patient to bear down? A. Uterus is fully perfused. B. Fetus is at -8 station. C. Cervix is completely dilated D. The patient feels ready.

C. Cervix is completely dilated Bearing down should occur only after the cervix is completely dilated. A term baby needs to be born despite uterus perfusion. There is no such thing as -8 station. It is not up to the patient to decide when to bear down.

A pregnant patient with injuries from a motor vehicle crash experiences uterine rupture of the endometrium, myometrium, and serosa. In which way should this patient's health status be documented? A. Emergency uterine rupture B. Partial uterine rupture C. Complete uterine rupture D. Incomplete uterine rupture

C. Complete uterine rupture A complete uterine rupture is one in which the endometrium, myometrium, and serosa have separated. An incomplete or partial rupture is one in which not all layers, but some, have been disrupted. There is no such thing as emergency rupture.

The nurse is preparing to perform Leopold maneuvers on a patient in labor. For which reason should the nurse complete this assessment? A. Measure fetal flexion B. Determine fetal age C. Determine fetal position D. Determine fetal size

C. Determine fetal position Leopold maneuvers are a systematic way to determine fetal position through palpating the maternal abdomen. Leopold maneuvers are not performed to determine fetal flexion, age, or size.

A postpartum patient's fundus is firm and located lower than the umbilicus. Which action should the nurse take? A. Massage the patient's abdomen. B. Provide the patient with sanitary pads. C. Document the finding. D. Administer 10 mg of oxytocin.

C. Document the finding. Because the fundus is expected to be firm at the umbilicus or lower and at midline, this finding should be documented. The uterus should be palpated, but not massaged unless boggy (atonic). The patient may need sanitary pads for the lochia, but not for a firm fundus. The patient does not require oxytocin therapy.

A patient in labor is diagnosed with premature placental separation. For which procedure should the nurse prepare the patient? A. Induced abortion B. Vacuum extraction C. Forceps-assisted delivery D. Cesarean birth

C. Forceps-assisted delivery Forceps-assisted delivery is indicated in premature placental separation. Vacuum extraction, cesarean birth, and induced abortion are not required for this patient.

A laboring patient requires an infusion of hypertonic glucose. For which condition should the nurse monitor the newborn? A. Hyperglycemia B. Hyperinsulinemia C. Hypoglycemia D. Hypothermia

C. Hypoglycemia When hypertonic glucose infusions are used during labor, there is an increase in maternal blood glucose, which can lead to hypoglycemia in the newborn. Before birth, the fetus can have hyperglycemia and hyperinsulinemia. Hypothermia is not related to glucose levels

A patient in labor receiving epidural analgesia is exhibiting signs of hypotension. Which action should the nurse make a priority? A. Place the patient into the supine position. B. Change the type of intravenous solution being infused. C. Increase the rate of intravenous fluid infusion. D. Place the patient in the reverse Trendelenburg position.

C. Increase the rate of intravenous fluid infusion. Hypotension is a result of spinal blockade, which lowers peripheral resistance and decreases cardiac output. Increasing intravenous fluids increases cardiac output and perfusion to the placenta. Placing the patient in the supine position does not increase perfusion to the placenta. Changing the type of intravenous solution being infused is not indicated in this situation. The reverse Trendelenburg position does not increase perfusion to the placenta and will worsen the state of hypotension.

The nurse is preparing teaching materials about abruptio placentae. Which information should the nurse include? A. Patients with diabetes mellitus are at higher risk. B. A patient with loop electrosurgical excision procedure (LEEP) of the cervix is at higher risk. C. It is more common in patients of advanced maternal age. D. It is seen more frequently in patients with lower economic status.

C. It is more common in patients of advanced maternal age. Abruptio placentae is more common in pregnancies complicated by advanced maternal age. Loop electrosurgical excision procedure (LEEP) of the cervix has been found to increase the risk of preterm birth. Preterm premature rupture of membranes (PPROM) is seen more frequently in patients with a lower economic status. Diabetes mellitus is a risk factor for bacteriuria.

A patient in labor has a prolapsed umbilical cord. In which position should the nurse place the patient? A. Lithotomy B. Supine C. Knee-chest D. Trendelenburg

C. Knee-chest The patient should be placed in a knee-chest position to relieve pressure on the prolapsed umbilical cord. The other positions do not relieve pressure on the cord and would not help the fetus.

The nurse is caring for a laboring patient whose fetus is in the occiput anterior position. Which outcome should the nurse expect for this patient? A. Emergency cesarean birth will likely be needed. B. The second stage of labor might be prolonged. C. Labor and birth are likely to proceed normally. D. The patient will experience strong backache and pelvic pressure.

C. Labor and birth are likely to proceed normally. Labor and birth are more likely to proceed normally when the fetus is in the occiput anterior position, which is the most common position. A malposition would indicate a likely need for cesarean birth. A patient might experience strong backache, pelvic pressure, and prolonged second stage of labor when the fetus is in a posterior position.

A patient in labor wants to avoid an episiotomy. Which birthing position should the nurse recommend for this patient? A. Recumbent B. Squatting C. Left lateral Sims D. Sitting on birthing stool

C. Left lateral Sims The left lateral Sims position increases perineal relaxation and decreases the need for an episiotomy. The recumbent, squatting, and sitting on birthing stool positions do not have this advantage.

A patient in labor is being prepared for delivery. Which action should the nurse take when positioning the patient in stirrups? A. Lift the right leg before the left. B. Ensure there is consistent pressure on the calves. C. Lift both legs simultaneously. D. Turn both straightened legs to either side

C. Lift both legs simultaneously. If stirrups are used, they should be padded to alleviate pressure, and both legs should be lifted simultaneously to avoid strain on abdominal, back, and perineal muscles. There should not be any pressure on the back of the knees or calves while the patient is in stirrups. Legs should be upright and not twisted to the side.

A patient in the third stage of labor requests an anesthetic. Which type of pain control should the nurse anticipate being prescribed? A. Lumbar spinal block B. Pudendal block C. Lumbar epidural block D. Local anesthetic

C. Lumbar epidural block A lumbar epidural block is placed within the epidural space and affects a larger area of nerves to include the uterus, cervix, vagina, and perineum. A lumbar spinal block relieves uterine pain only. A pudendal block relieves perineal pain only. Local infiltration affects the perineum only.

The nurse is preparing teaching materials on the advantages of analgesics during labor. Which advantage should the nurse associate with a spinal block? A. Long-acting B. Absence of maternal hypotension C. Maternal compartmentalization of the drug D. Ability to internally rotate the fetus

C. Maternal compartmentalization of the drug One of the advantages of spinal block is the maternal compartmentalization of the drug. Spinal block is short-acting. Absence of maternal hypotension is an advantage of a pudendal block. Internal rotation of the fetus can be accomplished with a lumbar epidural.

A patient is diagnosed with placenta accreta. For which health problem should the nurse monitor the patient? A. Macrosomia B. Prolapsed umbilical cord C. Maternal hemorrhage D. Polyhydramnios

C. Maternal hemorrhage In placenta accreta, the chorionic villi attach directly to the myometrium of the uterus, which can cause hemorrhage and death. Prolapsed umbilical cord, macrosomia, and polyhydramnios are not associated with placenta accreta.

The placenta of a pregnant patent has an accessory lobe of fetal villi. Which action should be the priority? A. Monitor the fetus for intrauterine growth restriction. B. Monitor the fetus for stress. C. Monitor the patient for hemorrhage. D. Monitor the fetus for prematurity.

C. Monitor the patient for hemorrhage. In succenturiate placenta, one or more accessory lobes of fetal villi develop on the placenta. Monitoring the patient for possible hemorrhage is the priority. Fetal prematurity is associated with circumvallate placenta and battledore placenta. Fetal intrauterine growth restriction is associated with circumvallate placenta. Fetal stress is associated with battledore placenta.

The nurse is planning care for a pregnant patient. For which reason should the nurse plan interventions to help prevent spontaneous preterm labor? A. Prolonged sitting B. History of cardiac disease C. Native American ancestry D. Maternal age of 30

C. Native American ancestry Patients of Native American ancestry are at risk for spontaneous preterm labor. A history of cardiac disease does not increase the risk for preterm labor. Maternal age of over 35 and prolonged standing increase the risk for preterm labor.

A postpartum patient receives a dose of morphine sulfate for pain. For which complication should the nurse monitor the patient? .A. Dizziness B. Sweaty, clammy skin C. Nausea D. Blurred vision

C. Nausea A common side effect of morphine sulfate is nausea. Dizziness is a common side effect of nitrous oxide. Sweaty, clammy skin is a common side effect of nalbuphine hydrochloride. Blurred vision is a common side effect of promethazine hydrochloride.

A patient being prepared for an external cephalic version has a transverse uterine scar on the lower abdomen. Which action should the nurse take? A. Prepare the patient for the version immediately. B. Administer 30 units of oxytocin. C. Notify the healthcare provider. D. Apply external fetal monitoring to the patient's abdomen.

C. Notify the healthcare provider. A lower uterine scar indicates that a previous cesarean birth occurred, which is a contraindication to external cephalic version. Prior scarring of the uterus may increase the risk of uterine tearing or uterine rupture, and should be reported to the healthcare provider. This patient should not have the version. An external fetal monitor is not required. Oxytocin is not indicated at this time.

The nurse is teaching a childbirth education class about fetal position during labor. Which position should the nurse explain is most likely to result in a normal delivery and birth? A. Posterior lateral B. Left-sacrum-transverse C. Occiput anterior D. Left-acromion-dorsal-posterior

C. Occiput anterior Labor and birth are more likely to proceed normally when the fetus is in the occiput anterior position. The other positions are malpositions.

The night nurse reviews the intake list of patients in labor and delivery. Which patient should the nurse assess first? A. Patient with cervical dilation of 4 centimeters requesting an epidural B. Patient with increasing back pain C. Patient with cervical dilation of 6 centimeters for 5 hours D. Patient with nausea and vomiting

C. Patient with cervical dilation of 6 centimeters for 5 hours Average cervical change in the active phase of the first stage of labor is 1.3 to 1.4 centimeters per hour; thus, the patient whose dilation remains unchanged for several hours requires further evaluation. A laboring patient's request for medication should be handled, but this is not the priority. Increasing back pain and nausea and vomiting are normal experiences during labor and do not require immediate assessment.

The nurse is caring for a patient in labor. For which reason should the nurse anticipate a forceps delivery? A. False labor B. Preterm pregnancy C. Postterm pregnancy D. Intense contraction

C. Postterm pregnancy Forceps may be required to help deliver a postterm fetus due to the baby's large size.Preterm babies are usually not large and would not require an assisted delivery. False labor and intense contractions do not require the use of forceps.

The fetus of a patient in labor is in transverse lie with shoulder presentation. Which action should the nurse take? A. Apply internal fetal scalp electrode. B. Monitor blood pressure every hour. C. Prepare for an emergency cesarean birth. D. Ambulate the patient every hour.

C. Prepare for an emergency cesarean birth. A fetus in transverse lie with shoulder presentation needs to be delivered immediately via cesarean birth. An internal fetal scalp electrode is applied when the face of the fetus is the presenting part. Ambulating the patient and monitoring blood pressure are not appropriate actions for the patient whose fetus is in transverse lie.

The nurse is caring for a patient in labor. For which situation should the nurse anticipate the use of vacuum extraction? A. Delayed first stage of labor B. Induction of labor C. Prolonged second stage of labor D. Fetus in transverse lie

C. Prolonged second stage of labor Vacuum extraction is indicated for a prolonged second stage of labor, especially if the patient is excessively fatigued. Labor induction would be needed if the first stage of labor is delayed. Cesarean birth is required for a fetus in transverse lie. Vacuum extraction is not necessarily needed during labor induction.

A patient who has been in labor for 1 hour starts to panic and demands a cesarean birth. Which action should the nurse take? A. Call the surgical staff. B. Administer an anxiolytic. C. Reassure the patient. D. Provide ice chips.

C. Reassure the patient. The patient in labor may have increased anxiety and feel out of control, which requires extra reassurance and support. The patient should be reassured that everything is progressing normally. The healthcare provider determines the need for a cesarean birth and prescribes antianxiety medication. Ice chips are unlikely to help with feelings of anxiety.

The nurse administers antibiotics to a patient in preterm labor. As which type of prevention strategy should the nurse document this action? A. Priority B. Tertiary C. Secondary D. Primary

C. Secondary Secondary prevention strategies for preterm labor include antibiotic therapy, which is used to prevent urinary and vaginal infections that occur during pregnancy. Administering antibiotics is not a primary prevention strategy. There is no such thing as a tertiary or priority prevention strategy.

The nurse is teaching a group of pregnant women about the effect of pelvic shape on childbirth. Which information should the nurse share? A. The android pelvis is most suitable for vaginal childbirth. B. The gynecoid pelvis is least suitable for vaginal childbirth. C. The anthropoid pelvis is most suitable for vaginal childbirth. D. The bifurcated pelvis is least suitable for vaginal childbirth.

C. The anthropoid pelvis is most suitable for vaginal childbirth. The most favorable pelvic types for vaginal childbirth are anthropoid and gynecoid. The android pelvis has a small diameter, which is not ideal for vaginal childbirth. There is no such thing as a bifurcated pelvis.

The nurse is caring for a patient who is experiencing preterm labor. Which outcome should the nurse include in the plan of care? A. The patient identifies two causes of preterm labor before discharge. B. The patient affirms that fears about early labor are nonexistent. C. The patient can identify signs and symptoms of preterm labor that need to be reported to the caregiver. D. The partner verbalizes how to take care of the patient during preterm labor.

C. The patient can identify signs and symptoms of preterm labor that need to be reported to the caregiver. It is important for the patient to identify the signs and symptoms of preterm labor that should be reported to the healthcare provider. It is not realistic to expect that the patient will experience no fear. Nursing outcomes should address the patient's self-care, not how the partner can care for the patient. The patient being able to identify two causes of preterm labor by discharge is a goal, not an outcome.

A patient who abuses opioids is in labor. Which should the nurse expect if an epidural is going to be used for pain management? A. The patient will likely not request an epidural until the last stage of labor. B. An epidural is contraindicated for the patient. C. The patient may experience higher levels of pain and desire an early epidural. D. An epidural would be less effective for the patient.

C. The patient may experience higher levels of pain and desire an early epidural. Because a patient with long-term opioid use may experience greater-than-average levels of pain, early epidural placement may be beneficial in ensuring the patient receives effective analgesia. Epidurals are not contraindicated or any less effective for patients with opioid dependence or addiction.

The nurse is developing a plan of care for a patient with premature preterm rupture of membranes (PPROM). Which outcome should the nurse identify as appropriate for this patient? A. The patient demonstrates how to correct a prolapsed cord. B. The patient's partner verbalizes three risk factors for PPROM. C. The patient's risk of prolapsed umbilical cord is decreased. D. The patient is free from infection during future pregnancies.

C. The patient's risk of prolapsed umbilical cord is decreased. Decreased risk of prolapsed umbilical cord is an appropriate outcome for this patient. Expecting the patient to be free of future infections does not focus on the current situation. The partner's ability to verbalize the risk factors for PPROM does not focus on the patient. The patient is not responsible for reinserting a prolapsed umbilical cord.

A patient with clinical depression is in labor. Which behavior should the nurse expect for this patient? A. Continual and excessive worrying B. Hallucinations and delusions C. Trouble concentrating D. Short-lived and intense feelings of nervousness

C. Trouble concentrating The patient with clinical depression who is in labor may exhibit trouble concentrating. Short-lived, unpredictable episodes of intense nervousness are associated with a panic disorder. Continual and excessive worrying is associated with generalized anxiety disorder. Hallucinations and delusions are associated with psychosis.

A pregnant patient's uterus is not equivalent with gestational age, and the fetus is easily palpated and not ballotable. For which procedure should the nurse prepare this patient? A. Amniocentesis B. Vaginal delivery C. Ultrasound D. Cesarean birth

C. Ultrasound Oligohydramnios may be occurring when the uterus does not increase in size according to established gestational dating and the fetus is easily palpated, outlined by the examiner, and not ballotable. When oligohydramnios is suspected, the fetus can be assessed by biophysical profile (BPP), nonstress tests (NSTs), and serial ultrasounds. Amniocentesis, vaginal delivery, and cesarean birth are not indicated for these symptoms.

A patient in labor is prescribed pain medication. At which time should the medication be given? A. After delivery of the newborn B. During the first phase of labor only C. When the nurse believes timing is right D. During the second phase of labor only

C. When the nurse believes timing is right In many organizations, the nurse decides when to give the analgesic agent prescribed by the healthcare provider. Analgesia after delivery is of no value to the patient. It is not restricted to the first or second phase of labor.

The nurse is teaching a pregnant patient requiring a vacuum extraction about a mediolateral episiotomy. Which patient statement should indicate the teaching was effective? A. "The cut will be made from my vagina to my rectum." B. "The episiotomy will be performed between contractions." C. "Repair will be done after birth but before the placenta is delivered." D. "A diagonal cut away from my vagina will be made."

D. "A diagonal cut away from my vagina will be made." The mediolateral incision begins in the middle of the vaginal opening and extends down toward the buttocks at a 45-degree angle. The midline episiotomy is a vertical incision that begins at the vaginal opening and extends downward toward the rectum. Episiotomy repair usually occurs after the placenta has been delivered in case a manual removal of the placenta or a uterine exploration is needed. An episiotomy is usually performed during a contraction, when the head is crowning.

The nurse is preparing a woman for childbirth by teaching the McRoberts position. Which statement should the nurse make? A. "Get on the floor on all four extremities and rock back and forth." B. "Squat, then straighten your legs while raising your arms above your head." C. "Stand with feet shoulder-width apart and turn side to side." D. "Lie on your back and move your knees to your shoulders."

D. "Lie on your back and move your knees to your shoulders." The McRoberts position is performed by flexing the patient's thighs toward the shoulders while lying on the back. The other statements do not describe the McRoberts position.

A patient learns about what to expect during the second stage of labor. Which patient statement indicates teaching was effective? A. "My vital signs will be assessed every 30 minutes." B. "My baby's heart rate will be evaluated with each contraction." C. "I should try to remain quiet and relaxed during labor." D. "My contractions will be continuously palpated."

D. "My contractions will be continuously palpated." During the second stage of labor, the patient's contractions will be continuously palpated. The patient's vital signs and fetal heart rate are monitored every 5 to 15 minutes. It is normal for a patient to be very vocal during labor and delivery.

A patient is admitted to the birthing unit. Which patient statement should the nurse note in the birth plan? A. "I am allergic to peanuts." B. "I understand that you offer only private rooms, which is what I want." C. "I am planning to breastfeed my newborn." D. "My husband will be my labor partner."

D. "My husband will be my labor partner." After admission to the birthing unit, the patient's birth plan and expectations for the delivery process should be acknowledged. Identifying who will be in the room during labor clarifies the patient's plans. The patient will not be eating a meal during labor and delivery, so the allergy is not important information at this time. The birthing unit only offers private rooms, so this preference does not need to be noted. Whether the patient plans to breastfeed or bottle-feed is not important to know during labor.

A patient in labor is encouraged to walk in the hallway as a distraction technique. Which action should the nurse take before the patient begins ambulation? A. Apply no-slip socks to the patient's feet. B. Administer pain medication. C. Encourage the partner to walk with the patient. D. Assess the fetal heart rate.

D. Assess the fetal heart rate. The fetal heart rate should be assessed before ambulation, since this activity can cause the fetal heart rate to increase. Wearing appropriate footwear is important but not the priority. Pain medication should be given after ambulation, as this could adversely affect the patient's safety. A partner can ambulate with the patient, but this is not a priority.

A patient who received an analgesic 30 minutes ago reports a need to urinate. Which action should the nurse take? A. Insert a catheter with a leg drainage bag. B. Tell the patient to wait for the nursing assistant. C. Place the bedside commode closer to the patient before leaving the room. D. Assist the patient to the toilet.

D. Assist the patient to the toilet. An analgesic administered 30 minutes earlier is taking effect on the patient. In order to maintain safety, the patient should be assisted to the toilet. Inserting a catheter is not appropriate or necessary during labor. The patient should not be expected to wait for assistance, as this could cause an injury if the patient attempts to ambulate alone. The use of a bedside commode requires assistance transferring to the device.

A patient's fetal heart rate ranges from 110 to 120 beats/min between contractions. In which way should the nurse interpret this information? A. Fetal bradycardia B. Fetal tachycardia C. Variable fetal heart rate D. Baseline fetal heart rate

D. Baseline fetal heart rate Baseline fetal heart rate (FHR) refers to the average FHR that is not associated with uterine contractions. The normal baseline rate ranges from 110 to 160 beats/min. Fetal bradycardia is a rate less than 110 beats/min during a 10-minute period or longer. Fetal tachycardia is a sustained rate greater than 160 beats/min. Fetal heart rate variability measures the activity of the sympathetic and parasympathetic nervous systems over a 10-minute period

A pregnant patient has blood trapped between the placenta and uterine wall. Which type of premature placental separation should the nurse document? A. Partial B. Marginal C. Complete D. Central

D. Central When the placenta separates centrally, the blood is trapped between the placenta and the uterine wall and results in concealed bleeding. In marginal separation, the blood passes between the fetal membranes and the uterine wall and escapes vaginally. Partial separation results in minimal to moderate bleeding. Complete separation is characterized by massive vaginal bleeding.

The nurse is preparing to assess fetal heart tones. For which fetal position should the heart tones be assessed on the lower maternal abdominal quadrant? A. Breech presentation B. Transverse lie C. Feet presentation D. Cephalic presentation

D. Cephalic presentation When the fetus is in a cephalic presentation, the fetal heart rate (FHR) is best heard in the lower quadrant of the maternal abdomen. For a fetus in a breech, or feet, presentation, the FHR is heard at or above the maternal umbilicus. For a fetus in a transverse lie, the FHR may be heard best just above or just below the maternal umbilicus.

A nulliparous 40-year-old pregnant patient is obese and close to term. For which potential procedure should the nurse prepare the patient? A. Vacuum extraction B. Amniocentesis C. Episiotomy D. Cesarean birth

D. Cesarean birth Nulliparity, high maternal age, and obesity are all risk factors for cesarean birth. There is no indication this patient may require a vacuum extraction, amniocentesis, or episiotomy.

A patient in labor experiences trickling of amniotic fluid during contractions. Which action should the nurse take? A. Decrease the epidural dose. B. Document the patient's pain level. C. Position the patient on the left side. D. Clean up the leaking fluid.

D. Clean up the leaking fluid. Leaking of amniotic fluid during contractions is a normal occurrence. The patient should be cleaned and made comfortable. There is no need to alter the epidural medication dose, assess for pain, or position on the left side as these actions will not affect the leaking of the fluid.

The nurse is assessing the fetus of a patient in labor. Which action should the nurse take when the fetal heart is located? A. Count for 30 seconds just after a contraction. B. Count for 15 seconds during a contraction. C. Count for 90 seconds before and during a contraction. D. Count for 60 seconds before, during, and just after a contraction.

D. Count for 60 seconds before, during, and just after a contraction. After the fetal heart rate is located, the rate should be assessed for 30 to 60 seconds to obtain the number of beats per minute. The rate should be assessed before, during, and just after a contraction to detect any abnormalities in heart rate.

The nurse observes a laboring patient's muscles starting to relax after a contraction. Which phase of contraction should the nurse understand this indicates? A. Increment B. Acme C. Beta D. Decrement

D. Decrement Decrement is the last phase of the contraction and occurs when the uterine muscles begin to relax. Increment is the building up of the contraction. Acme is the peak of the contraction. Beta is not a contraction phase.

The nurse is preparing teaching materials on creating a supportive environment for the patient in labor and family. Which action should the nurse recommend in this teaching? A. Spend minimal time with the patient to allow for rest. B. Provide the family with brochures about labor and delivery. C. Visit the birthing room every 10 minutes to evaluate the patient's comfort. D. Encourage questions when visiting the birthing room.

D. Encourage questions when visiting the birthing room. It is important for rapport to be established to create a supportive environment for the patient and family. One way to do this is to encourage the patient and family to ask questions at any time. Although the patient should be allowed to rest, this should not be done by limiting contact with the patient. It is not appropriate for the patient and family to learn about labor and birth while it is occurring. Comfort does not need to be assessed every 10 minutes.

An epidural increases the risk for complications, including the need for Pitocin to augment labor. Spinal, local infiltration, and pudendal analgesics do not increase the risk of needing Pitocin to augment labor. A. Recurrent variable decelerations B. Fetal heart rate 130 beats/min C. Moderate variability D. Frequent late decelerations

D. Frequent late decelerations Late decelerations are an abnormal finding that should caution against the use of analgesia in the patient in labor. Recurrent variable decelerations, a fetal heart rate between 110 and 160 beats/min, and moderate variability are normal findings that do not contraindicate administration of analgesia.

The placenta of a pregnant patient is partially covering the internal cervical os. Which grade of placenta previa should the nurse document? A. Grade 4 B. Grade 2 C. Grade 1 D. Grade 3

D. Grade 3 Grade 3 of placenta previa occurs when the placenta partially covers the internal cervical os. Grade 1 occurs when the placenta lies in the lower uterine segment, but its lower edge does not abut the internal cervical os. Grade 2 occurs when placental tissue reaches the margin of the internal cervical os, but does not cover it. Grade 4 occurs when the placenta completely covers the internal cervical os.

The nurse is preparing teaching materials on preterm premature rupture of membranes (PPROM). Which should the nurse identify as a common characteristic of patients with this health problem? A. Immigration B. Higher education C. Higher economic status D. Lower economic status

D. Lower economic status PPROM is more common in patients of lower economic status. Preterm labor is more common in educated patients of higher economic status. Immigration does not affect the occurrence of PPROM.

The nurse is preparing teaching on respiratory system changes during labor. Which finding should the nurse identify as normal and expected? A. A mild decrease in the respiratory rate as labor progresses B. Metabolic acidosis uncompensated by respiratory alkalosis in the first stage of labor C. Mild metabolic acidosis compensated by respiratory alkalosis after birth D. Mild respiratory acidosis in the beginning of the second stage of labor

D. Mild respiratory acidosis in the beginning of the second stage of labor As the patient enters the second stage of labor, PaCO2 levels may rise along with blood lactate levels (due to muscular activity), and mild respiratory acidosis occurs. A mild increase in the respiratory rate is normal in labor. Metabolic acidosis uncompensated by respiratory alkalosis occurs when the baby is born. Mild metabolic acidosis compensated by respiratory alkalosis occurs during the first stage of labor.

The nurse is caring for a newborn. In which position should the nurse place the newborn to ensure adequate respirations? A. Side-lying B. Semi-Fowler C. Prone D. Modified Trendelenburg

D. Modified Trendelenburg Because the first priority after birth is to maintain the newborn's respirations, the infant should be placed in the modified Trendelenburg position to aid drainage of mucus from the nasopharynx and trachea. The side-lying, semi-Fowler, and prone positions would not be used in this situation

A postpartum patient receiving pain medication reports nausea. Which medication should the nurse suspect the patient received? A. Promethazine hydrochloride B. Fentanyl (Sublimaze) C. Naloxone (Narcan) D. Morphine sulfate

D. Morphine sulfate Nausea is a common side effect of morphine sulfate. Nausea is not a side effect of promethazine hydrochloride or fentanyl. Naloxone is used to reverse the effects of narcotics and does not cause nausea.

A patient requires medication to induce labor. Which prescription should the nurse expect to administer? A. Dinoprostone inserted while patient is prone B. Dinoprostone inserted into the anterior portion of the vagina C. 50 mcg of misoprostol every 2 hours D. Oxytocin 4 hours after 50 mcg misoprostol

D. Oxytocin 4 hours after 50 mcg misoprostol Oxytocin should be administered no less than 4 hours after the last dose of misoprostol. Misoprostol administered every 2 hours is too frequent. After dinoprostone placement, the patient should be positioned supine. Dinoprostone should be inserted into the posterior portion of the vagina.

The nurse is caring for a laboring patient. Which finding should the nurse understand is normal and expected? A. Anuria B. Oliguria C. Nocturia D. Polyuria

D. Polyuria Polyuria is common during labor. It results from the increase in cardiac output, which causes an increase in the glomerular filtration rate and renal plasma flow. If anuria and oliguria occur during labor, the patient requires further evaluation. The patient will experience polyuria throughout the process and not just at night (nocturia).

A pregnant patient has the following outcome identified in the plan of care: "The patient's risk of prolapsed umbilical cord is decreased." For which health problem should this outcome most likely apply? A. Abruptio placentae B. Multiple gestation C. Placenta previa D. Premature preterm rupture of membranes (PPROM)

D. Premature preterm rupture of membranes (PPROM) Premature preterm rupture of membranes (PPROM) has many possible complications, including fetal hypoxia from cord prolapse or cord compression. A decreased risk of cord prolapse would be an expected outcome of nursing care for this health problem. Abruptio placentae, placenta previa, and multiple gestation do not cause prolapsed umbilical cord.

A pregnant patient arrives at the prenatal clinic for a routine appointment. For which reason should the nurse assess the patient's temperature immediately? A. Abruptio placentae B. Oligohydramnios C. Placenta previa D. Premature rupture of membranes (PROM)

D. Premature rupture of membranes (PROM) Because the rupture of membranes places the patient at risk for infection, temperature should be measured first. Abruptio placentae, placenta previa, and oligohydramnios do not cause infection (and thus fever).

A pregnant patient reports vaginal bleeding. Which laboratory value should indicate the patient is experiencing abruptio placentae? A. Fibrinogen levels greater than 150 mg/dL B. Platelet count less than 50,000 mm3 C. Partial thromboplastin time (PTT) longer than 90 seconds D. Prothrombin time (PT) longer than 15 seconds

D. Prothrombin time (PT) longer than 15 seconds Abruptio placentae is characterized by PT longer than 15 seconds, PTT longer than 60 seconds, fibrinogen levels less than 150 mg/dL, and platelet count less than 100,000 mm3.

A patient in labor is receiving an epidural for pain management. For which symptom should diphenhydramine be administered? A. Anxiety B. Constipation C. Nausea D. Pruritus

D. Pruritus Itching is a side effect of the medication used for an epidural infusion. An antihistamine such as diphenhydramine (Benadryl) can be administered to manage pruritus. The epidural should decrease the patient's anxiety. The use of diphenhydramine can cause gastrointestinal symptoms such as nausea.

A postpartum patient has a boggy uterus, dark vaginal bleeding, and blood clots. Which health problem should the nurse realize is occurring with this patient? A. Rupture of membranes B. Maternal infection C. Preterm labor D. Retained placental fragments

D. Retained placental fragments Retained placental fragments in the postpartum patient can lead to a boggy uterus, dark vaginal bleeding, and blood clots. Since the patient is postpartum, preterm labor and rupture of membranes would not be applicable. Maternal infection does not cause hemorrhage.

The nurse is preparing teaching on the use of systemic analgesics during labor. For which health problem should the nurse identify that nalbuphine hydrochloride (Nubain) is contraindicated? A. Glaucoma B. Convulsive disorder C. Acute asthma D. Sensitivity to sulfites

D. Sensitivity to sulfites Nalbuphine hydrochloride (Nubain) is contraindicated in patients with a sensitivity to sulfites. Promethazine hydrochloride (Phenergan) is contraindicated in those with glaucoma. Morphine sulfate is contraindicated in patients with convulsive disorders or acute asthma.

The nurse is educating a 13-year-old postpartum adolescent about newborn care. Which factor should the nurse consider when providing this teaching? A. The adolescent is primarily concerned with what her friends think. B. The adolescent has an attitude of "knowing it all." C. The adolescent is very stoic and quiet D. The adolescent has decreased problem-solving capabilities.

D. The adolescent has decreased problem-solving capabilities. Because cognitive development is incomplete in the younger adolescent, the patient may have fewer problem-solving capabilities. Younger adolescents are more concerned about what their parents think versus what their friends think. The older adolescent has an attitude of knowing it all. The middle adolescent is stoic.

The nurse is caring for a laboring patient whose fetus is not descending the birth canal in the presence of strong contractions. Which situation should the nurse understand is occurring? A. The fetus is stuck at +4 station. B. The fetus is not large enough for the birth canal. C. The fetus is in breech presentation. D. The fetal presenting part and maternal pelvis are disproportionate.

D. The fetal presenting part and maternal pelvis are disproportionate. Failure of the presenting part to descend in the presence of strong contractions may be due to disproportion between the maternal pelvis and fetal presenting part. A +4 station indicates imminent birth. Either the fetus is too large for the birth canal, or the pelvis is too narrow. There is no indication that the fetus is in breech position.

The nurse is identifying outcomes for a patient in the fourth stage of labor. Which outcome should the nurse include? A. The patient verbalizes three adverse effects of oxytocin. B. The patient utilizes effective breathing techniques. C. The newborn's Apgar score improves within one hour after birth. D. The newborn's physical well-being is supported.

D. The newborn's physical well-being is supported. An appropriate outcome for a patient in the fourth stage of labor is supporting the physical well-being of the newborn. Breathing techniques are used in the first and second stages of labor. Oxytocin is used to stimulate uterine contractions and is not needed in the fourth stage of labor. The newborn's Apgar score is assessed in the third stage of labor.

The nurse is preparing a patient for an emergency cesarean birth. Which outcome should the nurse include in the plan of care? A. The partner remains by the patient's side during delivery. B. The patient does not cry during labor and delivery. C. The patient ambulates the hallway every 15 minutes to promote labor. D. The patient experiences a sense of control regarding labor and delivery.

D. The patient experiences a sense of control regarding labor and delivery. An appropriate outcome for a patient having an emergency cesarean birth is for the patient to feel a sense of control, as there are many factors that are beyond the patient's control in this situation. It is acceptable for the patient to cry during labor and delivery, as this helps relieve tension. It may be important for the partner to remain by the patient's side during delivery, but this is not as important as empowering the patient to feel control. The patient who requires a cesarean birth will need to be sedated and cannot ambulate. This is also a goal rather than an outcome.

The nurse is caring for a laboring patient from a different culture who maintains a flat affect throughout the process. Which interpretation should the nurse make? A. The patient requires a psychiatric evaluation. B. The patient has been administered too much pain medication. C. The patient is not able to feel or process the pain. D. The patient stoically accepts pain.

D. The patient stoically accepts pain. Individuals tend to respond to painful stimuli in the way that is acceptable in their culture. In some cultures, it is natural to communicate pain, no matter how mild, whereas members of other cultures stoically accept pain out of fear or because it is expected. The patient's flat affect does not indicate that psychiatric evaluation is needed, too much medication was administered, or pain is not being felt.

A patient in labor has a fetal umbilical cord abnormality. Which outcome should the nurse identify for this patient's care? A. The patient will verbalize three causes of fetal umbilical cord abnormality. B. The patient will verbalize two position changes to prevent cord compression. C. The patient will remain in bed until delivery to prevent umbilical cord prolapse. D. The patient will not experience abnormal bleeding during labor and delivery.

D. The patient will not experience abnormal bleeding during labor and delivery. Since umbilical abnormalities may not be evident until delivery, the patient should be monitored closely to ensure no abnormal bleeding occurs. Verbalization of position changes or causes of umbilical cord abnormality are goals, not outcomes. There is no indication the patient is at risk for cord compression or umbilical cord prolapse, so outcomes for these health issues are not appropriate.

A pregnant patient is diagnosed with cephalopelvic disproportion. Which outcome should be identified to meet this patient's psychosocial needs? A. The patient verbalizes the desire to cry throughout labor and delivery. B. The patient does not experience anxiety about labor and delivery. C. The patient is able to remain stoic throughout labor and delivery. D. The patient's fear about labor and delivery is decreased.

D. The patient's fear about labor and delivery is decreased. An appropriate psychosocial outcome for the patient diagnosed with cephalopelvic disproportion is that the fear of delivery is decreased. It is unrealistic to expect no anxiety about labor and delivery, as all patients experience some amount of anxiety. Some patients may cry during a portion of the labor and delivery, but it is not normal for the patient to cry the entire time. Patients of some cultures may remain stoic during labor and delivery, but all patients should not be held to this standard.

A patient at 28 weeks' gestation is suspected of being in preterm labor. Which assessment finding should confirm the diagnosis? A. 10 contractions in a 60-minute period B. Cervical change of 0.5 cm per hour C. Cervical effacement of 50% D. Uterine contractions every 5 minutes for 20 minutes

D. Uterine contractions every 5 minutes for 20 minutes Preterm labor is defined as uterine contractions every 5 minutes for 20 minutes, or 8 contractions in a 60-minute period, in a patient between 20 and 36 6/7 weeks' gestation. Cervical dilation greater than 1 cm and cervical effacement of 80% also characterize preterm labor.


Ensembles d'études connexes

Written and verbal communication

View Set

Study Guide: Anatomy and Physiology: Digestive System

View Set

ATI Mobility & Tissue Integrity Quiz

View Set

U.S. History Texas Independence Chapter 9 quiz

View Set

Accounting Information Systems - C242

View Set

ATI Community Population, Public, and Global Health

View Set

Bio Aspects of Nutrition - Quiz 1

View Set